Вы находитесь на странице: 1из 138

X

2
=
-
Item 1 of 24
QuestiOn ld: 398 a'? Mark
<J
Previous
[>
Next
?
Tutorial
4l
Lab Values
~
Notes
~
Calculator
,
Reverse Color
aiiAJ
Text Zoom
0
3 •
4 Physical examination of a 23-year-old man diagnosed \'lith chronic hepatitis shows corneal changes that are v isible to the naked eye, as shown in
5 the image below.
6
7
8
9
10
11
12
13
14
15
16
17
18
19
20
21
22
23
24
This patient is most likely to have which of the following conditions?

Q A. Pulmonary emphysema

~ Ill ® 0
Block Time Rematntng: 00:00:08
TUTOR Feedback Suspend End Block

••
•• -·
~ ·
o· []
t

• ~
~--
I~ if A

11:31PM
t?i; ~·)) 2/12/2019 ~
2
3
4
5
6
7
8
9
10
11
12
13
14 This patient is most likely to have which of the following conditions?
15
16 O A. Pulmonary emphysema
17
18
0 B. Diabetes mellitus

19 0 C. Basal ganglia atrophy

20
21
0 D. Renal cancer

22 0 E. Hypertrophic cardiomyopathy

23 0 F. Congenital deafness
24

Submit
2
3
4
5
6
7
8
9
10
11
12
13
14 This patient is most likely to have which of the following conditions?
15
16 A. Pulmonary emphysema [2%]
17
B. Diabetes mellitus (14%]
18
C. Basal ganglia atrophy [68%]
19
20 D. Renal cancer [3%]
21
E. Hypertrophic cardiomyopathy [7%]
22
23 F. Congenital deafness (3%]
24

Omitted
1011512018
X

2
=
-
Item 1 of 24
QuestiOn ld: 398 a'? Mark
<J
Previous
[>
Next
?
Tutorial
4l
Lab Values
~
Notes
~
Calculator
,
Reverse Color
aiiAJ
Text Zoom
0
3
4
Explanation
5
6
The image demonstrates a Kayser-Fleischer ring, an ophthalmologic finding most strongly associated with Wilson's disease (although it can also
7
be seen in chronic cholestatic diseases such as primary biliary cirrhosis). Kayser-Fleischer rings are best identified on slit-lamp examination, but
8
they can be observed grossly, especially with long-standing, untreated disease. The rings are formed through the granular deposition of copper
9 within Oescemers membrane in the cornea.
10
Wilson's disease is a rare, autosomal recessive disease most often identified in individuals 5-40 years old. The genetic mutations associated with
11
Wilson's disease hinder copper metabolism by reducing the formation and secretion of ceruloplasmin and decreasing hepatic secretion of copper
12
into the biliary system. Copper is a pro-oxidant and causes damage to hepatic tissue through the generation of free radicals. Eventually, it leaks
13
from injured hepatocytes into the circulation to be deposited in various tissues, including the cornea and basal ganglia. Atrophy of the basal
14
ganglia then ensues.
15
16 Treatment is life-long and focuses on removing accumulated copper in the tissues and preventing its re-accumulation. First-line medications
17 include copper chelators such as d-penicillamine and trientine.

18 (Choice A) Pulmonary emphysema commonly develops in patients w ith alpha-1 antitrypsin deficiency.
19
(Choice B) Diabetes mellitus commonly develops in patients with advanced hemochromatosis. It has no association with Wilson's disease.
20
21 (Choice D) Renal cancer (clear cell carcinoma) occurs more frequently in patients with von Hippei-Lindau disease, not Wilson's disease.
22
(Choice E) Hypertrophic cardiomyopathy presenting in childhood and adolescence is an autosomal dominant disease involving the cardiac
23 sarcomere. It is not associated with Wilson's disease.
24
(Choice F) Congenital deafness can develop secondary to numerous hereditary and nonhereditary cond itions. Wilson's disease has no
association with congenital deafness.

Educational objective:

~ Ill ® 0
Block Time Remammg: 00:00:13
TUTOR Feedback Suspend End Block

••
•• -·
~ ·
o· []
t

• ~
~--
I~ if A

11:31PM
t?i; ~·)) 2/12/2019 ~
X

2
=
-
Item 1 of 24
QuestiOn ld: 398 a'? Mark
<J
Previous
[>
Next
?
Tutoria l
4l
Lab Values
~
Notes
~
Calculator
,
Reverse Color
aiiAJ
Text Zoom
0
3
be seen in chronic cholestatic diseases such as pri mary biliary cirrhosis). Kayser-Fleischer rings are best identified on slit-lamp examination, but
4
they can be observed grossly, especially with long-standing, untreated disease. The rings are formed through the granular deposition of copper
5
within Descemet's membrane in the cornea.
6
7 Wilson's disease is a rare, autosomal recessive disease most often identified in individuals 5-40 years old. The genetic mutations associated with
8 Wilson's disease hinder copper metabolism by reducing the formation and secretion of ceruloplasmin and decreasing hepatic secretion of copper

9 into the biliary system. Copper is a pro-oxidant and causes damage to hepatic tissue through the generation of free radicals. Eventually, it leaks

10
from injured hepatocytes into the circulation to be deposited in various tissues, including the cornea and basal ganglia. Atrophy of the basal
ganglia then ensues.
11
12 Treatment is life-long and focuses on removing accumulated copper in the tissues and preventing its re-accumulation. First-line medications
13 include copper chelators such as d-penicillamine and trientine.
14
(Choice A) Pulmonary emphysema commonly develops in patients with alpha-1 antitrypsin deficiency.
15
16 (Choice B) Diabetes mellitus commonly develops in patients with advanced hemochromatosis. It has no association with Wilson's disease.

17 (Choice D) Renal cancer (clear cell carcinoma) occurs more frequently in patients with von Hippei-Lindau disease, not Wilson's disease.
18
(Choice E) Hypertrophic cardiomyopathy presenting in childhood and adolescence is an autosomal dominant disease involving the cardiac
19
sarcomere. It is not associated with Wilson's disease.
20
21 (Choice F) Congenital deafness can develop secondary to numerous hereditary and nonhereditary cond itions. Wilson's disease has no
22 association with congenital deafness.

23 Educational objective :
24 The Kayser-Fleischer ring is an ophthalmologic finding most strongly associated with Wilson's disease. It is seen most freq uently in patients with
neuropsychiatric complications. Basal ganglia atrophy is typically present in these patients.

Copyright® UWOI1d All ngllts reserved


X
1
=
-
ltem2of24
QuestiOn ld: 1740 a'? Mark
<J
Previous
[>
Next
?
Tutorial
4l
Lab Values
~
Notes
~
Calculator
,
Reverse Color
aiiAJ
Text Zoom
0
3
4 A 4-hour-old girl with a cleft lip is breastfeeding without difficulty. The infant was born vaginally after an uncomplicated pregnancy and delivery.
5 Routine antenatal sonography at 20 weeks gestation showed no abnormalities. Physical examination shows an intact palate and a unilateral d eft
6 lip on the left side. No other abnormalities are seen. This child's d eft lip resulted from failure of which of the following intrauterine processes?
7
8
0 A. Fusion of the 2 medial nasal prominences

9 0 B. Fusion of the maxillary prominence and intermaxillary segment


10
11
0 C. Fusion of the palatal processes

12 0 D. Hyperplasia of the frontonasal prominence


13
0 E. Hypoplasia of the mandibular prominence
14
15
16
Submit
17
18
19
20
21
22
23
24

~ Ill ® 0
Block Time Rematntng: 00:00:14
TUTOR Feedback Suspend End Block

••
•• -·
~ ·
o· []
t

• ~
~·-
I~ if A

11:31PM
t?i; ~·)) 2/12/20 19 ~
X
1
= ltem2of24 <J [> ? &I ~ ~ , ail!) 0
- QuestiOn ld: 1740 a'? Mark Previous Next Tutoria l Lab Values Notes Calculator Reverse Color Text Zoom
3 •
4 A 4-hour-old girl with a cleft lip is breastfeeding without difficulty. The infant was born vaginally after an uncomplicated pregnancy and delivery.
5 Routine antenatal sonography at 20 weeks gestation showed no abnormalities. Physical examination shows an intact palate and a unilateral d eft
6 lip on the left side. No other abnormalities are seen. This child's d eft lip resulted from failure of which of the following intrauterine processes?
7
A. Fusion of the 2 medial nasal prominences [33%)
8
9 B. Fusion of the maxillary prominence and intermaxillary segment (59%)
10
C. Fusion of the palatal processes (5%)
11
12 D. Hyperplasia of the frontonasal prominence [1%)
13
E. Hypoplasia of the mandibular prominence [ 1%)
14
15
16
17 G ttled llo. fi\ 4 seconds
59% ~ 10/09/2018
Correct answer l!.!!.
18 Answered correcUy \.:.; Time Spent EE3 last Updated
B
19
20
21 Explanation
22
23 Normal facial development
X
1
= ltem2of24 <J [> ? &I ~ ~ , ail!) 0
- QuestiOn ld: 1740 a'? Mark Previous Next Tutoria l Lab Values Notes Calculator Reverse Color Text Zoom
3
4
Normal facial development
5
6
7
8
9
10
11
12
13
14
15
16
17
Week 2 Week S WeekS
18
19
0 frontonasal prominence maxillary prominence

20 lateral nasal prominence mandibular prominence

21 medial nasal prominence stomodeum


22 e uworld
23 The lip and palate form during the fifth-sixth week of embryonic development through a series of fusions;
24
1. The first pharyngeal arch splits into the upper maxillary prominence and the lower mandibular prominence.
2. Fusion of the 2 medial nasal prominences forms the midline intermaxillary segment. The intermaxillary segment will become the philtrum
of the upper lip, the 4 medial maxillary teeth, and the primary palate.
3. The left and then fuse with the midline intE!rmaxillary ~~~
X
1
=
-
ltem2of24
QuestiOn ld: 1740 a'? Mark
<J
Previous
[>
Next
?
Tutoria l
4l
Lab Values
~
Notes
~
Calculator
,
Reverse Color
aiiAJ
Text Zoom
0
3
The lip and palate form during the fifth-sixth week of embryonic development through a series of fusions:
4
5 1. The first pharyngeal arch splits into the upper maxillary prominence and the lower mandibular prominence.
6 2. Fusion of the 2 medial nasal prominences forms the midline intermaxillary segment. The intermaxillary segment will become the philtrum
7 of the upper lip, the 4 medial maxillary teeth, and the primary palate.
8 3. The left and right maxillary prominences then fuse with the midline intermaxillary segment to form the upper lip and primary palate. If one
9 of the maxillary prominences fails to fuse with the intermaxillary segment, a unilateral cleft lip results. If both maxillary prominences fail to
10 fuse with the intermaxillary segment, bilateral d eft lip results.

11 (Choice A) Failure of the medial nasal prominence to fuse and form the intermaxillary segment is associated with severe midline defeds, such as
12 holoprosencephaly.
13
(Choice C) During the seventh-eighth week of embryonic development, the maxillary prominences give rise to palatine shelves. The thin sheets
14
of tissue that comprise the palatine shelves grow medially and fuse into the secondary palate. Fusion of the secondary palate with the posterior
15
aspect of the primary palate forms the complete palate. Failure of any of these processes can lead to cleft palate. The presence of a cleft lip
16
increases the risk of cleft palate, but either can occur in isolation.
17
18 {Choice D) The orbits normally arise from the sides of the face and rotate medially. However, excessive tissue from the frontal nasal prominence
could result in orbital hypertelorism (wide-set eyes) in addition to a broad forehead and wide nasal bridge.
19
20 {Choice E) Micrognathia is characterized by hypoplasia of the mandibular prominence. In Pierre-Robin sequence, the severe micrognathia
21 results in posterior displacement of the tongue (glossoptosis) and prevents fusion of the secondary palate (cleft palate).
22
Educational objective :
23
Cleft lip results when the maxillary promi nence fails to fuse with the intermaxillary segment during the fifth-sixth week of embryonic development.
24
Cleft palate occurs when the palatine shelves fail to fuse with one another or with the primary palate. Cleft lip and palate can occur together or in
isolation.
1

3
4
5 Orofaclal clefts
6
7
8
9
10
11
12
13
14
IsOlated unilaumol cleft liP ISOlated bilateral cleft palate Unilateral dell tip and
15 anterior deft palate

16
17
18
19
20
21
22
23
24 Undateral c1en 11;> Normal palate
and trilaleral cleft palate

Zoom In Zoom Out Reset Add To Flash Card

~ IJI ® 0
Block lime Remaining : 00:00:17
TUTOR Feedback Suspend End Block

•• -o·
_,· lfi w -• rGJ rc:"~
11:32 PM
•• IJ9 •• >f A ~? x ~
ft?. c:J>l) 2/12/2019 ~
X
1
2
=
-
ltem3of24
QuestiOn ld: 1646 a'? Mark
<J
Previous
[>
Next
?
Tutoria l
4l
Lab Values
~
Notes
~
Calculator
,
Reverse Color
aiiAJ 0
Text Zoom


4 A 43-year-old man comes to the emergency department due to painless, progressive visual impairment. The patient was diagnosed with sexually
5 acquired HIV infection 4 years ago. He was previously started on antiretroviral therapy but has been noncompliant with his prescribed
6 medications. His most recent CD4 count was 37 cellslmm3 . Physical examination shows no skin rash or corneal lesions. Funduscopy reveals the
7 findings seen in the image below.
8
9
10
11
12
13
14
15
16
17
18
19
20
21
22
23
24
1
2

4
5
6
7
8
9
10
11
12
13
14 Which of the following medications is the best initial therapy for this patient?
15
16 O A. Acyclovir
17
18
0 B. Amphotericin B

19 0 C. Clarithromycin

20
21
0 D. Flucytosine

22 0 E. Ganciclovir

23 0 F. Penicillin G
24

Submit
1
2

4
5
6
7
8
9
10
11
12
13
14 Which of the following medications is the best initial therapy for this patient?
15
16 A. Acyclovir [10%1
17
B. Amphotericin B [6%1
18
C. Clarithromycin [3%1
19
20 D. Flucytosine (3%]
21
E. Ganciclovir [7 4%1
22
23 F. Penicillin G (1%1
24

Omitted
01109/2019
X
1
2
=
-
ltem3of24
QuestiOn ld: 1646 a'? Mark
<J
Previous
[>
Next
?
Tutorial
4l
Lab Values
~
Notes
~
Calculator
,
Reverse Color
aiiAJ
Text Zoom
0

4
5
6
7
8
9
10
11
12

'
13
14
15
16
'
17
18
19
20
21
22
23
24
Patients with untreated AIDS who have CD4 counts <50/mm3 are at high risk for cytomegalovirus (CMV) retinitis. Suspicion is often raised
when a patient with advanced AIDS develops slowly progressive blurred vision, scotoma (blind spots), floaters, or photopsia (flashing lights).
Although most cases are thought to arise due to hematogenous dissemination of CMV, serum testing for the virus is often (-50%) negative.

~ Ill ® 0
Block Time Rematntng: 00:00:24
TUTOR Feedback Suspend End Block

••
•• -·
~ ·
o· []
t

• ~
~--
I~ if A

11:32 PM
t?i; ~·)) 2/12/2019 ~
1
2

4
Patients with untreated AIDS who have CD4 counts <50/mm3 are at high risk for cytomegalovirus (C MV) r etinitis . Suspicion is often raised
5
when a patient with advanced AIDS develops slowly progressive blurred vision, scotoma (blind spots), floaters, or photopsia (flashing lights).
6
Although most cases are thought to arise due to hematogenous dissemination of CMV, serum testing for the virus is often (-50%) negative.
7
Therefore, the diagnosis is typically established when funduscopy reveals y ellow-w hite, fluffy retinal lesions near the retinal vessels associated
8
with hemorrhage.
9
10 Treatment is requ1red to prevent progression and blindness. Ganciclovir, a guanine nucleoside analogue that is structurally similar to acyclovir

11
but expresses greater activity against CMV DNA polymerase, is first-line therapy. Patients are also typically started on antiretroviral therapy to
increase CD4 count A major complication of CMV retinitis is retinal detachment due to the replacement of damaged retina with thin, atrophic scar
12
tissue (which is prone to tear).
13
14 (Choice A) Acyclovir is used to treat herpes simplex (HSV) and varicella zoster virus 0fZV) infections. Although these infections occasionally
15 cause acute retinal necrosis, most cases arise in older, immunocompetent individuals (not patients with advanced AIDS). In addition, HSV retinitis
16 and VZV retinitis are usually associated with significant vitreal inflammation, spotty peripheral retinal lesions, and infrequent hemorrhage_
17
(Choice B ) Candida endophthalmitis is treated with amphotericin B. Funduscopy usually reveals focal, white, mound-like lesions on the retina.
18 Most cases occur in the setting of an indwelling central venous catheter or total parenteral nutrition.
19
(Choice C) Clarithromycin is used in conjunction with other antibiotics for the treatment of Mycobacterium avium complex infection, which usually
20
causes fever, night sweats, abdominal pain, and diarrhea.
21
22 (Choice D) Cryptococcus neoformans infections are treated with flucytosine, a fluorinated analog of cytosine, in combination w ith amphotericin
23 B. Although cryptococcal endophthalmitis can occur in patients with advanced AIDS, it is much less common than CMV retinitis, and patients
24 often have other manifestations of cryptococcal infection such as skin lesions or meningoencephalitis.

(Choice F) Penicillin G is used to treat ocular (and neuro-) syphilis. Ocular syphilis often presents as uveitis but can cause retinitis, which usually
appears as ground-glass lesions with significant vitreal inflammation. Most patients also have manifestations of syphilitic meningitis (eg,
headache, vomiting).

~ Ill ® 0
Block Time Rematntng: 00:00:24
TUTOR Feedback Suspend End Block

••
•• -·
~ ·
o· []
t

• ~
~·-
I~ if A

11:32 PM
t?i; ~·)) 2/12/2019 ~
X
1
2
=
-
ltem3of24
QuestiOn ld: 1646 a'? Mark
<J
Previous
[>
Next
?
Tutoria l
4l
Lab Values
~
Notes
~
Calculator
,
Reverse Color
aiiAJ 0
Text Zoom

Treatment is required to prevent progression and blindness. Gancicl ovir, a guanine nudeoside analogue that is structurally similar to acydovir
4
but expresses greater activity against CMV DNA polymerase, is first-line therapy. Patients are also typically started on antiretroviral therapy to
5
increase CD4 count. A major complication of CMV retinitis is retinal detachment due to the replacement of damaged retina with thin, atrophic scar
6 tissue (which is prone to tear).
7
(Choice A) Acydovir is used to treat herpes simplex (HSV) and varicella zoster virus (VN) infections. Although these infections occasionally
8
cause acute retinal necrosis, most cases arise in older, immunocompetent individuals (not patients with advanced AIDS). In addition, HSV retinitis
9
and VZV retinitis are usually associated with significant vitreal inflammation, spotty peripheral retinal lesions, and infrequent hemorrhage.
10
11 (Choice B) Candida endophthalmitis is treated with amphotericin B. Funduscopy usually reveals focal, white, mound-like lesions on the retina.
12 Most cases occur in the setting of an indWelling central venous catheter or total parenteral nutrition.
13
(Choice C) Clarithromycin is used in conjunction with other antibiotics for the treatment of Mycobacterium avium complex infection, which usually
14 causes fever, night sweats, abdominal pain, and diarrhea.
15
(Choice D) Cryptococcus neoformans infections are treated with flucytosine, a fluorinated analog of cytosine, in combination ¥lith amphotericin
16
B. Although cryptococcal endophthalmitis can occur in patients with advanced AIDS, it is much less common than CMV retinitis, and patients
17
often have other manifestations of cryptococcal infection such as skin lesions or meningoencephalitis.
18
19 (Choice F) Penicillin G is used to treat ocular (and neuro-) syphilis. Ocular syphilis often presents as uveitis but can cause retinitis, which usually
20 appears as ground-glass lesions with significant vitreal inflammation. Most patients also have manifestations of syphilitic meni ngitis (eg,
21 headache, vomiting ).

22 Educational objective :
23 Cytomegalovirus retinitis is the most common cause of ocular disease in patients with untreated AIDS who have CD4 counts <50/mm3 . Diagnosis
24 is made by funduscopy, which typically reveals yellow-white, fluffy retinal lesions near the retinal vessels with associated hemorrhage. Treatment
with ganciclovir is required to prevent blindness.

Copyright® UWOI1d All ngllts reserved


X
1
2
=
-
ltem4of24
QuestiOn ld: 8589 a'? Mark
<J
Previous
[>
Next
?
Tutorial
4l
Lab Values
~
Notes
~
Calculator
,
Reverse Color
aiiAJ
Text Zoom
0
• 3
A 54-year-old woman comes to the clinic due to difficulty hearing for the past few weeks. During the neurologic examination , the physician
5 assesses her hearing using a vibrating tuning fork. The handle of the tuning fork is placed on her left mastoid process until the sound is no longer
6 audible. The tines are then quickly placed near the patient's left auditory meatus, and she reports hearing no sound. When the handle of the
7 vibrating fori< is placed on the middle of her forehead, she hears the vibration more strongly in her left ear than her right. This patient is most likely
8 experiencing which of the following types of hearing loss?
9
10
O A. Conductive loss in both ears

11 0 B. Conductive loss in left ear


12
13
0 c. Conductive loss in right ear

14 0 D. Sensorineural loss in both ears


15
16
0 E. Sensorineural loss in left ear

17 0 F. Sensorineural loss in right ear

18
19
20 Submit

21
22
23
24

~ Ill ® 0
Block Time Rematntng: 00:00:25
TUTOR Feedback Suspend End Block

••
•• -·
~ ·
o· []
t

• ~
~

I~
..
if A

11:32 PM
t?i; ~·)) 2/12/2019 ~
X
1
= ltem4of24 <J [> ? 4l ~ ~ , ail!) 0
2 - QuestiOn ld: 8589 a'? Mark Previous Next Tutorial Lab Values Notes Calculator Reverse Color Text Zoom
• 3 •
A 54-year-old woman comes to the clinic due to difficulty hearing for the past few weeks. During the neurologic examination , the physician
5 assesses her hearing using a vibrating tuning fork. The handle of the tuning fork is placed on her left mastoid process until the sound is no longer
6 audible. The tines are then quickly placed near the patient's left auditory meatus, and she reports hearing no sound. When the handle of the
7 vibrating fork is placed on the middle of her forehead, she hears the vibration more strongly in her left ear than her right. This patient is most likely
8 experiencing which of the following types of hearing loss?
9
A. Conductive loss in both ears [0%)
10
11 B. Conductive loss in left ear [68%)
12
C. Conductive loss in right ear [5%)
13
14 D. Sensorineural loss in both ears [1%)
15 E. Sensorineural loss in left ear [20%)
16
17
F. Sensorineural loss in right ear [3%)

18
19
20 Omtlted
Correct answer
llo. 68% IT\ 3 Seconds ~ 11 /07/2018
21 L!.!!. Answered correcUy \.::.J Time Spent EE3 Last Updated
B
22
23
24 Explanation

Interpretation of Weber & Rinne tests

~ Ill ® 0
Block Time Remaonong: 00:00:27
TUTOR Feedback Suspend End Block

••
•• o_,·
~ ·

[]
t

• ~
~

·~
..
If A ~· ri?. ~·))
11:32 PM
2/12/2019 ~
X
1
= ltem4of24 <J [> ? &I ~ ~ , ail!) 0
2 - QuestiOn ld: 8589 a'? Mark Previous Next Tutorial Lab Values Notes Calculator Reverse Color Text Zoom
• 3

Interpretation of Weber & Rinne tests


5
6
Rinne result Weber result
7
8
No rmal Midline
9
10 AC >BC bilaterally
Sensorineural
11 Lateralizes to unaffected ear
hearing loss
12
13
Conductive BC >AC in affected ear,
14 Lateralizes to affected ear
hearing loss AC >BC in unaffected ear
15
AC = air conduction; BC = bone conduction.
16
~UWorld
17
18 Hearing loss is classified as either conductive (impaired transmission of air vibrations to inner ear) or sensorineural (involving the cochlea or
19 auditory nerve). The Rinne and Weber tests can be used to help determine the type of hearing loss.
20 The Rinne test compares air versus bone conduction (via the mastoid bone). As the vibration of the tuning fork fades, air-conducted sound is
21 normally louder and heard longer than bone-conducted sound. The Rinne test is considered positive (normal) if the sound is heard best at the
22 external auditory meatus (air conduction) and negative (abnormal) if the patient hears the vibration better at the mastoid (bone conduction).
23
• Bone conduction greater than air conduction suggests conductive hearing loss.
24
The Weber test is performed by placing a vibrating tuning for1< on the middle of forehead equidistant from both ears. The vibration carried by bone
conduction is normally heard equally in both ears; vibration heard louder in one ear is abnormal.

• Conductive hearing loss causes lateralization to the affected ear as the conduction deficit masks the ambient noise in the room, allowing the

~ Ill ® 0
Block Time Rematntng: 00:00:27
TUTOR Feedback Suspend End Block

••
•• _,
o• []
t

• ~
~--

·~ If A ~x
11:32 PM
!I;; ~·)) 2/12/2019 ~
X
1
2
= ltem4of24 <J [> ? &I ~ ~ , ail!) 0
- Questoon ld: 8589 Ia \='Mark Previous Next Tut o ria l Lab Values Notes Calculator Reverse Color Text Zoom
... • • • - - - # ... .

• 3 Laterahzes to affected ear


hearing loss AC >BC in unaffected ear

5 AC = air conduction; BC = bone conduction.


6 OUWor1d
7 Hearing loss is classified as either conductive (impaired transmission of air vibrations to inner ear) or sensorineural (invoMng the cochlea or
8 auditory nerve). The Rinne and Weber tests can be used to help determine the type of hearing loss.
9
The Rinne test compares air versus bone conduction (via the mastoid bone). As the vibration of the tuning fork fades, air-conducted sound is
10
normally louder and heard longer than bone-conducted sound. The Rinne test is considered positive (normal) if the sound is heard best at the
11
external auditory meatus (air conduction) and negative (abnormal) if the patient hears the vibration better at the mastoid (bone conduction).
12
13 • Bone conduction greater than air conduction suggests conductive hearing loss.
14 The Weber test is performed by placing a vibrating tuning fork on the middle of forehead equidistant from both ears. The vibration carried by bone
15 conduction is normally heard equally in both ears; vibration heard louder in one ear is abnormal.
16
• Conductive hearing loss causes lateralization to the affected ear as the conduction deficit masks the ambient noise in the room, allowing the
17
vibration to be better heard.
18
• Sensorineural hearing loss causes lateralization to the unaffected ear as the unimpaired inner ear can better sense the vibration.
19
20 The Ri nne test is abnormal in this patient's left ear, and the Weber test lateralizes to her left ear. These findi ngs suggest conductive hearing
21 loss in the left ear.

22 Educational objective :
23 In conductive hearing loss, bone cond uction will be greater than air conduction (abnormal Rinne test), and the Weber test will lateralize to the
24 affected ear. In sensorineural hearing loss, air conduction will be greater than bone conduction (normal Rinne test), and the Weber test will
lateralize to the unaffected ear.

Copyright® UWOI1d All ngllts reserved


X
1
2
=
-
ltem5of24
QuestiOn ld: 8.63 a'? Mark
<J
Previous
[>
Next
?
Tutorial
4l
Lab Values
~
Notes
~
Calculator
,
Reverse Color
aiiAJ
Text Zoom
0
3
4 A 3-month-old boy is brought to the office by his mother for a routine checkup. He is found to have a right-sided white pupillary reflex. His father
was diagnosed with a retinal neoplasm during childhood and had his left eye enudeated. This child is at the greatest risk of developing which of
6 the following neoplasms?
7
8 O A. Acute lymphoblastic leukemia

9
0 B. Neuroblastoma
10
11 0 C. Ewing sarcoma

12 0 D. Osteosarcoma
13
14
0 E. Medulloblastoma

15
16
Submit
17
18
19
20
21
22
23
24

~ Ill ® 0
Block Time Rematntng: 00:00:29
TUTOR Feedback Suspend End Block

••
•• -·
~ ·
o· []
t

• ~
~--
I~ if A

11:33 PM
t?i; ~·)) 2/12/2019 ~
X
1
2
=
-
ltem5of24
QuestiOn ld: 8.63 a'? Mark
<J
Previous
[>
Next
?
Tutorial
4l
Lab Values
~
Notes
~
Calculator
,
Reverse Color
aiiAJ
Text Zoom
0
3 •
4 A 3-month-old boy is brought to the office by his mother for a routine checkup. He is found to have a right-sided white pupillary reflex. His father
was diagnosed with a retinal neoplasm during childhood and had his left eye enudeated. This child is at the greatest risk of developing which of
6 the following neoplasms?
7
A. Acute lymphoblastic leukemia [2%)
8
9 B. Neuroblastoma [25%)
10
C. Ewing sarcoma [5%)
11
12 D. Osteosarcoma [59%)
13
E. Medulloblastoma [7%)
14
15
16
17 G ttled llo. fi\ 3 seconds
59% ~ 12/ 19/2018
Correct answer l!.!!.
18 Answered correcUy \.:.; Time Spent EE3 last Updated
0
19
20
21 Explanation
22
23 Retinoblastoma is the most common ocular tumor of childhood. It usually presents with a white pupillary reflex (leukocoria) in children <5 years
24 old. Approximately 60% of retinoblastomas are sporadic (usually unilateral}, while 40% are familial (often, but not always, bilateral). Children with
sporadic retinoblastoma are not at risk for other malignancies, while patients with familial retinoblastoma often develop other tumors, usually
sarcomas, later in life. Among these secondary malignancies, osteosarcoma (Choice D) occurs most often.

Familial retinoblastoma occurs due to a germline mutation that affects the Rb tumor suppressor gene on chromosome 13. All cells in the body of

~ Ill ® 0
Block Time Remaonong: 00:00:30
TUTOR Feedback Suspend End Block

••
•• -·
~ ·
o· []
t

• ~
~--
I~ if A

11:33 PM
t?i; ~·)) 2/12/2019 ~
1
2
3
old. Approximately 60% of retinoblastomas are sporadic (usually unilateral), while 40% are familial (often , but not always, bilateral). Children with
sporadic retinoblastoma are not at risk for other malignancies, while patients 1'1ith familial retinoblastoma often develop other tumors, usually
sarcomas, later in life. Among these secondary malignancies, osteosarcoma (Choice D) occurs most often.
6
7 FamiliaI retinoblastoma occurs due to a germline mutation that affects the Rb tumor suppressor gene on chromosome 13. All cells in the body of
8 an affected individual carry this mutation. As there are two alleles of the Rb gene (one from each parent), the malignancy does not occur while the
9 second allele is functional. A somatic second mutation (occurring in the retinal cells only) acquired early in life is the "second hir that leads to
10 malignancy. The ceU with two inactivated Rb genes becomes the source of the malignant tumor. Sporadic retinoblastomas occur due to two
11 acquired somatic mutations affecting only the retinal cell lineage.

12
13
Knudson's 2-Hit Hypothesis
14
15 Both copies of the gene must be knocked out in order to promote malignancy
16
17
18
19
Sporadic cancer:
20
21
2 acquired mutations .. ..
22
23 Tumor
24
X
1
= ltem5of24 <J [> ? &I ~ ~ , ail!) 0
2 - QuestiOn ld: 8.63 a'? Mark Previous Next Tutorial Lab Values Notes Calculator Reverse Color Text Zoom
3
4
Knudson's 2-Hit Hypothesis

6
Both copies of the gene must be knocked out in order to promote malignancy
7
8
9
10
Sporadic cancer:
11
12
2 acquired mutations
• • •
13
14 Tumor
15
16
17
18
19 Hereditary cancer:
20
21
1 inherited and
1 acquired mutation
• •
22
23 Tumor
24
© UWorld

The Rb gene is a tumor suppressor; the corresponding protein exists in an active and inactive state. The active (dephosphorylated) Rb protein
does not allow the cell to proceed from G1 to the S stage of the cell cycle. When the cell is stimulated by a growth factor, the Rb protein is
1
2
3
Hereditary cancer:
1 inherited and II II
1 acquired mutation

7
Tumor
8
9 ©UWorld
10
The Rb gene is a tumor suppressor; the corresponding protein exists in an active and inactive state. The active (dephosphorylated) Rb protein
11
does not allow the cell to proceed from G1 to the S stage of the cell cyde. When the cell is stimulated by a growth factor, the Rb protein is
12
phosphorylated and converted into the inactive state. Inactivation of the Rb protein permits cell division. Cells with two inactive Rb genes divide
13
uncontrollably and give rise to malignancy.
14
15 (Choice A) Patients with Down syndrome, ataxia-telangiectasia, and neurofibromatosis type 1 have an increased risk of development of acute

16
lymphoblastic leukemia, which is not associated with familial retinoblastoma.

17 (Choice B) Neuroblastoma is a common childhood cancer that arises from sympathetic nervous tissue, most commonly in the adrenals. Most
18 cases are sporadic.
19
(Choice C) Ewing sarcoma is an undifferentiated neoplasm of neuroectodermal cells that most commonly occurs in the long and flat bones of
20
children and adolescents. It occurs sporadically.
21
22 (Choice E) Medulloblastoma is the most common malignant brain tumor of childhood, typically arising in the cerebellum .

23 Educational objective:
24 Familial retinoblastoma occurs as a result of mutations of each of the two Rb genes ("two hits"). These patients have an increased risk of
secondary tumors, especially osteosarcomas, later in life.

Copyright® UWOI1d All ngllts reserved


X
1
2
=
-
ltem6of24
Questoon ld: 8703 Ia \='Mark
<J
Previous
[>
Next
?
Tutorial
4l
Lab Values
~
Notes
~
Calculator
,
Reverse Color
aiiAJ
Text Zoom
0
3
4 A 23-year-old man comes to the emergency department complaining that he has food stuck in his throat. His symptoms started 2 hours ago after
5 eating fish at a local seafood restaurant. He has tried coughing and swallowing multiple times in an attempt to clear the food, but has so far been
unsuccessful. The patient denies any difficulty with breathing. He does not appear to be in any distress on physical examination. Laryngoscopy
7 reveals a fish bone lodged in the left piriform recess. During an attempt to retrieve the fish bone, a nerve is injured deep to the mucosa overlying
8 the recess. Which of the following is most likely to be impaired in this patient?
9
10
O A. Cough reflex

11 0 B. Gag reflex
12
13
0 c. Mastication

14 0 D. Salivation
15
16
0 E. Taste sensation

17
18
Submit
19
20
21
22
23
24

~ Ill ® 0
Block Time Remaonong: 00:00:32
TUTOR Feedback Suspend End Block

••
•• -·
~ ·
o· []
t

• ~
~--
I~ if A

11:33 PM
t?i; ~·)) 2/12/2019 ~
X
1
2
= ltem6of24 <J [> ? &I ~ ~ , ail!) 0
- Questoon ld: 8703 Ia \='Mark Previous Next Tut oria l Lab Values N otes Calculator Reverse Color Text Zoom
3 •
4 A 23-year-old man comes to the emergency department complaining that he has food stuck in his throat. His symptoms started 2 hours ago after
5 eating fish at a local seafood restaurant. He has tried coughing and swallowing multiple times in an attempt to clear the food, but has so far been
unsuccessful. The patient denies any difficulty with breathing. He does not appear to be in any distress on physical examination. Laryngoscopy
7 reveals a fish bone lodged in the left piriform recess. During an attempt to retrieve the fish bone, a nerve is injured deep to the mucosa overlying
8 the recess. Which of the following is most likely to be impaired in this patient?
9
A. Cough reflex [36%)
10
11 B. Gag reflex [54%)
12
C. Mastication (1%)
13
14 D. Salivation [2%)
15 E. Taste sensation (4%]
16
17
18
Om1tted
19
Correct answer
llo. 36% fi\ 4 Seconds ~ 09/29/2018
20 l!!.!. Answered correcUy \.::J Time Spent EE3 Last Updated
A
21
22
23 Explanation
24
Posterior view of pharynx
X
1
2
= ltem6of24 <J [> ? &I ~ ~ , ail!) 0
- Questoon ld: 8703 Ia \='Mark Previous Next Tutorial Lab Values Notes Calculator Reverse Color Text zoom
3
4
Posterior view of pharynx
5
~
1
2
3
4
5 Epiglottis
Internal branch o f
7 superior laryngea l
8 nerve
9 Mucosa overlying
10 internal laryngeal - -s - -Thyroid cartilage
11 nerve
12
13
Piriform recess
14
15
16
17
18
19
20
21
22 © UWorld
23 The piriform recesses are small cavities that lie on either side of the laryngeal orifice. They are bounded medially by the aryepiglottic folds and
24 laterally by the thyroid cartilage and thyrohyoid membrane. During normal swallowing, food is diverted by the epiglottis laterally through the
piriform recesses into the esophagus without endangering the airway.

A thin layer of mucosa overlying the piriform recess is all that protects the superficially coursing internal laryngeal nerve, a branch of the superior
Unlike the recurrent and external ~i!)fl~
1
2
3
4
5
© UWortd

7
The piriform recesses are small cavities that lie on either side of the laryngeal orifice. They are bounded medially by the aryepiglottic folds and
laterally by the thyroid cartilage and thyrohyoid membrane. During normal swallowing, food is diverted by the epiglottis laterally through the
8
piriform recesses into the esophagus without endangering the airway.
9
10 A thin layer of mucosa overlying the piriform recess is all that protects the superficially coursing internal laryngeal nerve, a branch of the superior
11 laryngeal nerve (CN X). Unlike the recurrent and external laryngeal nerves that carry motor fibers to the musdes involved in vocal cord function,
12 the internal laryngeal nerve contains only sensory and autonomic fibers. It mediates the afferent limb of the cough reflex by carrying sensation
13 from the mucosa superior to the vocal cords. Foreign bodies, such as chicken or fish bones, can become lodged in the piriform recess. The
14 internal laryngeal nerve can be damaged by the sharpness of these objects or attempts to retrieve them.

15 (Choice B ) The afferent limb of the gag reflex is mediated predominantly by the glossopharyngeal nerve (CN IX), while the efferent limb is
16 carried by the vagus nerve (CN X). The internal laryngeal nerve does not carry motor fibers and mediates sensation mainly from larynx and
17 epiglottis (stimulation of these areas induces coughing, not gagging).
18
(Choice D) Salivation is mediated in part by parasympathetic fibers originating from the glossopharyngeal nerve. These fibers synapse on the
19
otic ganglion, and postganglionic fibers travel via the au riculotemporal nerve to reach the parotid gland.
20
21 (Choice E) Taste from the base of the tongue is mediated by the glossopharyngeal nerve (CN IX}; taste from the anterior two-thirds of the tongue
22 is med iated by the facial nerve (CN VII).

23 Educational objective :
24 The internal laryngeal nerve mediates the afferent limb of the cough reflex above the vocal cords. Foreign bodies (eg, chicken or fish bones) can
become lodged in the piriform recess and may cause damage to the nerve, impairing the cough reflex.

Copyright® UWOI1d All ngllts reserved


1
2
3
4
5
CriiniiJ nerve feflexes
• 6 'I
7 ~llox Elftct All-• Elf-·

8
9
10
-
~la<yllght

O:wnoal-x
Ught-induced pupi
constrltbon

Comul snmuttoon GlUM'S


orbaArlsoaA to <OOIOiiO

V1
(ophtbalmrc
II

'Ill
11 oyolids branch)

12
13
.._,._, S<ri!<dl-~
mosseter tor1tiOO>CNl
113
(Sensocylrom V1
muKio 51>-<1

__ '-·-
14
C~<\'0
15
16
Vestibulo-
eclAir .,_
moo;tmeftl:J We Pf'OIIIde
'Ill
N~IV. VI
..e..c head/body-
c_.,..,u
17
18 X
(arotld SinUS - caro<id """'
~·-
.,!let prasureau.es- IX
19 bean ra~ &bbxt pressure- •lsoinhlbll»d)

20 c...rgh Induced by Upp<f


Cough,_ X
21 repir.ttoty tract initarion

22 Rltfler conti'Xtion ol the


p~ constricu>t
23 Gog ..llox IX X
mUKJ<s-...bytoudung
24 tho bad< of tho duoot

Zoom In Zoom Out Reset A dd To Flash Card

~ IJI ® 0
Block Time Remaining : 00:00:34
TUTOR Feedback Suspend End Block

•• -·
_, lfi w -• rGJ rc:"~
11:33 PM
•• o· IJ9 •• >f A ~? x ~
ft?. c:J>l) 2/12/2019 ~
X
1
2
=
-
ltem7of24
Questoon ld: 11675 Ia \='Mark
<J
Previous
[>
Next
?
Tutorial
4l
Lab Values
~
Notes
~
Calculator
,
Reverse Color
aiiAJ
Text Zoom
0
3 •
4 A 62-year-old man with a history of poor medical follow-up comes to the office due to difficulty seeing. He has had blurry vision for the last 2
5 weeks. A day ago, he noticed a shadow develop suddenly across the visual field of his right eye. The patient has had no recent headaches,
6 double vision, vert1go, light sensitivity, or nausea. On examination, pupils are equal and reactive to light. Visual field testing by confrontation
reveals patchy loss of vision. A flame-shaped retinal hemorrhage in the right eye is noted on funduscopic examination, as shown in the image
8 below.
9
10
11
12
13
14
15
16
17
18
19
20
21
22
23
24

Which of the following is the most likely cause of his visual disturbance?

~ Ill ® 0
Block Time Rematntng: 00:00:38
TUTOR Feedback Suspend End Block

••
•• -·
~ ·
o· []
t

• ~
~--
I~ if A

11:33 PM
t?i; ~·)) 2/12/2019 ~
1
2
3
4
5
6

8
9
10
11 Which of the following is the most likely cause of his visual disturbance?
12
13 O A Central retinal artery occlusion
14
15
0 B. Cigarette smoking

16 0 C. Closed-angle glaucoma
17
18
0 D. Giant cell arteritis

19 0 E. Hyperlipidemia

20
0 F. Hypertension
21
22 0 G. Migraine

23 0 H. Thyroid eye disease


24

Submit
1
2
3
4
5
6

8
9
10
11 Which of the following is the most likely cause of his visual disturbance?
12
13 A Central retinal artery occlusion (18%]
14
B. Cigarette smoking (4%]
15
16 C. Closed-angle glaucoma [4%]
17 D. Giant cell arteritis [3%]
18
19
E. Hyperlipidemia [3%]

20 F. Hypertension [66%]
21
G. Migraine (0%]
22
23 H. Thyroid eye disease (0%]
24

Omitted
12105/2018
X
1
2
=
-
ltem7of24
Questoon ld: 11675 Ia \='Mark
<J
Previous
[>
Next
?
Tutorial
4l
Lab Values
~
N ot es
~
Calculator
,
Reverse Color
aiiAJ
Text Zoom
0
3
4
5
6

8
9
10
11
12
13
14
15
16
17
18
19
20
21
This patient has an acute retinal hemorrhage (arrow), most likely caused by severe, poorly controlled hypertension. Hypertensive retinal
22
hemorrhage typically causes painless, unilateral visual disturbances, ranging from mild obscuration without loss of visual acuity to permanent
23
blindness. Severe hypertension in retinal precapillary arterioles causes endothelial disruption, leakage of plasma into the arteriolar wall , and
24
fibrinous necrosis. The necrotic vessels can then bleed into the nerve fiber layers, causing dot- and flame-shaped hemorrhages.

Diagnosis of hypertensive retinal hemorrhage is usually confinned by direct ophthalmoscopic evaluation. Other findings of hypertensive
retinopathy include thickening of the arteriolar walls ("copper or silver wiring"), compression of the associated veins (arteriovenous nicking}, and
X
1
2
=
-
ltem7of24
Questoon ld: 11675 Ia \='Mark
<J
Previous
[>
Next
?
Tutorial
4l
Lab Values
~
Notes
~
Calculator
,
Reverse Color
aiiAJ
Text Zoom
0
- -- -
3
4 This patient has an acute retinal hemorrhage (arrow), most likely caused by severe, poorly controlled hypertension. Hypertensive retinal

5 hemorrhage typically causes painless, unilateral visual disturbances, ranging from mild obscuration without loss of visual acuity to permanent

6
blindness. Severe hypertension in retinal precapillary arterioles causes endothelial disruption, leakage of plasma into the arteriolar wall, and
fibrinous necrosis. The necrotic vessels can then bleed into the nerve fiber layers, causing dot- and flame-shaped hemorrhages.

8 Diagnosis of hypertensive retinal hemorrhage is usually confirmed by direct ophthalmoscopic evaluation. Other findings of hypertensive
9 retinopathy indude thickening of the arteriolar walls ("copper or silver wiring"), compression of the associated veins (arteriovenous nicking), and
10 small, white foci of retinal ischemia (cotton-wool spots).
11
(Choices A and 0) Central retinal artery ocdusion is most commonly due to atherosclerosis, cardioembolic disease, or vasculitis (eg , giant cell
12 arteritis). It causes acute monocular vision loss. Examination can reveal a chenry red spot at the macula due to diffuse retinal ischemia, but
13 hemorrhage is not typically seen.
14
(Choices B and E) Smoking and hyperlipidemia can increase the risk of developing ischemic retinal and optic vascular disease; however, these
15
are not associated with an increased risk of retinal hemorrhage.
16
17 (Choice C) Closed-angle glaucoma is characterized by increased intraocular pressure from impaired aqueous humor drainage. It causes acute
18 headache, eye pain, vision loss, and an afferent pupillary defect. Ophthalmoscopic examination reveals cupping of the optic disc without
19 hemorrhage.
20 {Choice G) Ocular migraine and migrai ne with visual aura can present with transient visual disturbances, with or without headache.
21 Ophthalmoscopic examination is normal in patients with migraine.
22
{Choice H) Thyroid eye disease (eg , Graves disease) can present with diplopia and proptosis; however, it is not associated with an increased risk
23
of retinal hemorrhage.
24
Educational objective:
Severe hypertension in retinal precapillary arterioles causes endothelial disruption, leakage of plasma into the arteriolar wall, and fibrinous
necrosis. The necrotic vessels can then bleed into the nerve fiber layer, which can be seen on examination as dot- or flame-shaped hemorrhages.

~ Ill ® 0
Block Time Rematntng: 00:00:43
TUTOR Feedback Suspend End Block

••
•• -·
~ ·
o· e mw []
t

• ~
~--
I~ if A ~·
11::\4 PM
t?i; ~·)) 2/12/20 19 ~
1
2
3
4
5
6

8
9
10
11
12
13
14
15
16
17
18
19
20
21
22
23
24

Zoom In Zoom Out Reset Add To Flash Card

~ IJI ® 0
Block Time Remaining : 00:00:43
TUTOR Feedback Suspend End Block

•• -o·
_,· lfi w -• rGJ rc:"~
11 :34 PM
•• IJ9 •• >f A ~? x ~
ft?. <l>l) 2/12/2019 ~
X
1
2
=
-
ltem8of24
QuestiOn ld: 1814 a'? Mark
<J
Previous
[>
Next
?
Tutorial
4l
Lab Values
~
Notes
~
Calculator
,
Reverse Color
aiiAJ
Text Zoom
0
3
4 A 15-year-old boy comes to the office due to right ear itching and discomfort for the past several days. He has no fever or hearing loss but has
5 noted scant drainage of thin, w hitish fluid. The patient has been taking swimming lessons at a local gym. On examination, there is no redness
6 around the ear, but gentle traction of the pinna elidts pain. During inspection of the ex1emal auditory canal, a speculum is inserted into the meatus
7 in dose contact with its posterior wall , causing the patient to suddenly become lightheaded and faint. He recovers spontaneously within a few
minutes with no residual confusion. Which of the following nerves was most likely irritated during the procedure?
9
10
O A. Accessory

11 0 B. Facial
12
13
0 c. Trigeminal

14 0 D. Vagus
15
16
0 E. Vestibulocochlear

17
18
Submit
19
20
21
22
23
24

~ Ill ® 0
Block Time Remammg: 00:00:44
TUTOR Feedback Suspend End Block

••
•• -·
~ ·
o· e mw []
t

• ~
~·-
I~ if A ~·
11::\4 PM
t?i; ~·)) 2/12/20 19 ~
X
1
= ltem8of24 <J [> ? &I ~ ~ , ail!) 0
2 - QuestiOn ld: 1814 a'? Mark Previous Next Tutorial Lab Values Notes Calculator Reverse Color Text Zoom
3 •
4 A 15-year-old boy comes to the office due to right ear itching and discomfort for the past several days. He has no fever or hearing loss but has
5 noted scant drainage of thin, w hitish fluid. The patient has been taking swimming lessons at a local gym. On examination, there is no redness
6 around the ear, but gentle traction of the pinna elidts pain. During inspection of the ex1emal auditory canal, a speculum is inserted into the meatus
7 in dose contact with its posterior wall , causing the patient to suddenly become lightheaded and faint. He recovers spontaneously within a few
minutes with no residual confusion. Which of the following nerves was most likely irritated during the procedure?
9
A. Accessory [0%)
10
11 B. Facial [10%)
12
C. Trigeminal [6%]
13
14 D. Vagus [50%)
15 E. Vestibulocochlear [32%]
16
17
18
Omttted
19 llo. 50% fi\ 3 Seconds ~ 12/05/2018
Correct answer l!!.!.
20 Answered correcUy \.::J Time Spent EE3 Last Updated
0
21
22
23 Explanation
24
Cutaneous innervation of the ear

~ Ill ® 0
Block Time Remaonong: 00:00:46
TUTOR Feedback Suspend End Block

••
•• _,
o• e mw []
t

• ~
~--

·~ If- A ~x
11::\4 PM
!I;; ~·)) 2/12/2019 ~
X
1
2
= ltem8of24 <J [> ? &I ~ ~ , ail!) 0
- Questoon ld: 1814 Ia \='Mark Previous Next Tutorial Lab Values Notes Calculator Reverse Color Text zoom
3
Cutaneous innervation of the ear
4
5
6
7

9
10
Branch of Facial
11
(CN VII)
12
13
• Auricular branch of Vagus
14 (CN X)
15
16 • Auriculotemporal
17 (CN V3 ·Trigeminal)
18
19 Lesser occipital
20 (C2, C3)
21
22 • Great auricular
(C2, C3)
23
24

~ Ill ® 0
Block Time Rematntng: 00:00:46
TUTOR Feedback Suspend End Block

••
•• _,
o• e mw []
t

• ~
~--

·~ If A ~x
11::\4 PM
!I;; ~·)) 2/12/2019 ~
1
2
3
Branch of Facial
4
(CN VI I)
5
6
• Auricular branch of Vagus
7
Q~-1:---- (CN X)

9 Auriculotemporal
10 t=::'r----::~"71:;:...._--+--- (CN V3 -Trigeminal)

11
12
13
14
15 • Great auricular
16 (C2, C3)
17
18
19
20
21
22
23
@UWMd
24
The majority of the external ear receives cutaneous innervation from the great auricular nerve, lesser occipital nerve, and auriculotemporal nerve.
Most of the external auditory canal, including the external portion of the tympanic membrane, is innervated by the mandibular division of the
trigeminal nerve (cranial nerve (CNJ V3) via its auriculotemporal branch.

~ Ill ® 0
Block Time Rematntng: 00:00:46
TUTOR Feedback Suspend End Block

••
•• _,
o• e mw []
t

• ~
~--

·~ If A ~x
11 ::\4 PM
!I;; ~·)) 2/12/2019 ~
~
1
2
3
4
5
6
7 CDUWO<td

The majority of the external ear receives cutaneous innervation from the great auricular nerve, lesser occipital nerve, and auriculotemporal nerve.
9 Most of the external auditory canal, induding the external portion of the tympanic membrane, is innervated by the mandibular division of the
10 trigeminal nerve (cranial nerve [CN] V3) via its auriculotemporal branch.
11
However, the posterior part of the external auditory canal, as well as the concavity and posterior eminentia of the concha, is innervated by the
12
small auricular branch of the vagus nerve (CN X). This patient has experienced vasovagal syncope after stimulation of his posterior external
13
auditory canal by an otoscope speculum. In this form of syncope, parasympathetic outflow via the vagus nerve leads to decreased heart rate and
14
blood pressure.
15
16 (Choice A) The accessory nerve (CN XI) provides voluntary motor innervation to the trapezius and the sternodeidomastoid.

17 (Choice B) The facial nerve (CN VII) has a small motor component to the stapedius and muscles behind the ear. It supplies cutaneous sensation
18 to a small area of the auricle, but stimulation here would be unlikely to trigger a vasovagal response.
19
(Choice C) The mandibular branch of the trigeminal nerve carries sensation from the majority of the external auditory meatus, except for the
20
posterior wall.
21
22 (Choice E) The vestibulocochlear nerve (CN VIII) mediates hearing and vestibular proprioception. It does not med iate any cutaneous sensation.

23 Educational objective:
24 The vagus nerve provides cutaneous sensation to the posterior external auditory canal via its small auricular branch. Sensation to the rest of the
canal is from the mandibular division of the trigeminal nerve.

Copyright® UWOI1d All ngllts reserved

~ Ill ® 0
Block Time Remammg: 00:00:46
TUTOR Feedback Suspend End Block

••
•• _,
o• e mw []
t

• ~
~--

·~ If A ~x
11::\4 PM
!I;; ~·)) 2/12/20 19 ~
X
1
2
=
-
ltem9of24
Questoon ld: 12257 Ia \='Mark
<J
Previous
[>
Next
?
Tutorial
4l
Lab Values
~
Notes
~
Calculator
,
Reverse Color
aiiAJ 0
Text Zoom
3 •
4 A 31-year-old woman comes to the office due to transient visual changes that are characterized by "loss of vision for a minute" and are commonly
5 precipitated by bending forward or lifting objects. She has had persistent headaches over the last 3 months that she attributes to migraines. The
6 patient takes over-the-counter analgesics as needed. Blood pressure is 140/90 mm Hg and pulse is 72/min and regular. BMI is 32.4 kg/rn2.
7 Funduscopic examination of both eyes reveals the findings shown in the image below.
8

10
11
12
13
14
15
16
17
18
19
20
21
22
23
24
X
1
2
=
-
ltem9of24
Questoon ld: 12257 Ia \='Mark
<J
Previous
[>
Next
?
Tutorial
&I
Lab Values
~
Notes
~
Calculator
,
Reverse Color
aiiAJ 0
Text zoom
3
4
5
6
7
8

10
11
12
13
14
15
16
17
18
19
20
21
22
23
24

Which of the following is the most likely cause of the observed funduscopic findings in this patient?

~ Ill ® 0
Block Time Rematntng: 00:00:52
TUTOR Feedback Suspend End Block

••
•• -·
~ ·
o· []
t

• ~
~--
I~ if A

11::\4 PM
t?i; ~·)) 2/12/2019 ~
1
2
3
4
5
6
7
8

10
11
12
13
14
15
Which of the following is the most likely cause of the observed funduscopic findings in this patient?
16
17
18
O A. Choroidal inflammation

19 0 B. Decreased arterial blood flow

20
0 c. Elevated intraocular pressure
21
22 0 D. Impaired optic nerve axoplasmic flow

23 0 E. Optic nerve inflammation


24

Submit
1
2
3
4
5
6
7
8

10
11
12
13
14
15
Which of the following is the most likely cause of the observed funduscopic findings in this patient?
16
17 A. Choroidal inflammation [3%)
18
B. Decreased arterial blood flow [21%)
19
20 C. Elevated intraocular pressure [50%)
21
D. Impaired optic nerve axoplasmic flow [14%)
22
23 E. Optic nerve inflammation (9%]
24

Omitted
0910612018
X
1
2
=
-
ltem9of24
Questoon ld: 12257 Ia \='Mark
<J
Previous
[>
Next
?
Tutorial
4l
Lab Values
~
Notes
~
Calculator
,
Reverse Color
aiiAJ
Text Zoom
0
3
4
5
6
7
8

10
11
12
13
14
15
16

17
18
19
20
21
22
23

24

This patient's clinical presentation is consistent with idiopathic intracranial hypertension (pseudotumor cerebri). This condition typically
in young obese women with daily headache, bilaterally symmetric and transient visual disturbances related to im1paired

~ Ill ® 0
Block Time Rematntng: 00:00:56
TUTOR Feedback Suspend End Block

••
•• -·
~ ·
o· e mw []
t

• ~
~·-
I~ if A ~·
11::\4 PM
t?i; ~·)) 2/12/2019 ~
1
2
3
4
5 This patient's clinical presentation is consistent with idiopathic intracranial hypertension (pseudotumor cerebri). This condition typically
6 presents in young obese women with daily headache, bilaterally symmetric papilledema, and transie nt visual disturbances related to impaired
7 cerebral venous outflow and elevated intracranial pressure. Symptoms characteristically worsen during Valsalva (eg, bending down, coughing) as
8 intracranial pressure increases.

Increased intracranial pressure is transmitted through the cerebrospinal fluid in the subarachnoid space, which is continuous with the optic nerve
10 sheath. This buildup of pressure compresses the optic nerves externally, which in tum impairs axoplasmic flow within the optic nerves, causing
11 bilateral optic disc edema (papilledema). Compared to normal funduscopy, patients with papilledema have elevation of the optic disc with
12 blurred disc margins (black arrows).
13
(Choice A) Choroidal inflammation is characteristic of posterior uveitis, which typically presents with painless vision loss and floaters. Opthalmic
14
examination may show inflammation/leukocytes in the vitreous humor. Uveitis is often associated w ith systemic inflammatory disorders (eg,
15
inflammatory bowel disease, ankylosing spondylitis).
16
17 (Choice B) Retinal ischemia can manifest with painless transient monocular vision loss (amaurosis fugax) and is most commonly caused by

18
atherosclerotic emboli originating from the ipsilateral carotid artery. Funduscopy may show embolic plaques and retinal whitening (due to
ischemia) . This condition typically affects older patients with vascular risk factors (eg, hypertension, hyperlipidemia, diabetes mellitus).
19
20 (Choice C) An acute increase in intraocular pressure is characteristic of angle-closure glaucoma, which typically presents with painful monocular
21 vision loss, headache, vomiting, and conjunctival injection w ith a poorly reactive mid-dilated pupil. Open-angle glaucoma results in prog ressive,
22 painless, bilateral vision loss w ith optic disc cupping on funduscopy (due to optic disc atrophy). Both of these conditions usually affect older
23 individuals.
24
(Choice E) Optic neuritis is freq uently associated with multiple sclerosis, a demyelinating central nervous system disease that typically affects
women age 15-50. Optic neuritis causes monocular vision loss over several weeks with painful eye movement. Patients can have an afferent
pupillary defect with hyperemia and swelling of the optic disc on funduscopy.
1
2
3
4 examination may show inflammation/leukocytes in the vitreous humor. Uveitis is often associated with systemic inflammatory disorders (eg,

5
inflammatory bowel disease, ankylosing spondylitis).

6 (Choice B) Retinal ischemia can manifest with painless transient monocular vision loss (amaurosis fugax) and is most commonly caused by
7 atherosclerotic emboli originating from the ipsilateral carotid artery. Funduscopy may show embolic plaques and retinal whitening (due to
8 ischemia). This condition typically affects older patients with vascular risk factors (eg, hypertension, hypertipidemia, diabetes mellitus).

(Choice C) An acute increase in intraocular pressure is characteristic of angle-closure glaucoma, which typically presents with painful monocular
10
vision loss, headache, vomiting, and conjunctival injection with a poorty reactive mid-dilated pupil. Open-angle glaucoma results in progressive,
11
painless, bilateral vision loss with optic disc cupping on funduscopy (due to optic disc atrophy). Both of these conditions usually affect older
12 individuals.
13
{Choice E) Optic neuritis is frequently associated with multiple sclerosis, a demyelinating central nervous system disease that typically affects
14
women age 15-50. Optic neuritis causes monocular vision loss over several weeks with painful eye movement. Patients can have an afferent
15
pupillary defect with hyperemia and swelling of the optic disc on funduscopy.
16
17 Educational objective:
18 Idiopathic intracranial hypertension (pseudotumor cerebri) presents in young obese women with daily headache (which worsens during Valsalva),
19 bilaterally symmetric papilledema, and transient visual disturbances. Increased intracranial pressure compresses the optic nerves, resulting in
20 impaired axoplasmic flow and optic disc edema.

21
22 References
23
• PathogenesiS of opllc disc edema in raised intracranial pressure.
24
• Pseudotumor cerebn bnef review of clinical syndrome and imaging findings.

Copyright® UWOI1d All ngllts reserved

~ Ill ® 0
Block Time Rematntng: 00:00:56
TUTOR Feedback Suspend End Block

••
•• -·
~ ·
o· e mw []
t

• ~
~--
I~ if A ~·
1LJS PM
t?i; ~·)) 2/12/2019 ~
1
2
3
4
5
6
7
8

10
11
12
13
14
15
16
17
18
19
20
21
22
23
24

Zoom In Zoom Out Reset


X
1
2
=
-
Item 10of24
Questoon ld: 11628 Ia \='Mark
<J
Previous
[>
Next
?
Tutorial
4l
Lab Values
~
Notes
~
Calculator
,
Reverse Color
aiiAJ 0
Text Zoom
3
4 A 48-year-old woman comes to the office due to an intermittent ear discharge over the last 2 years. She has also noticed decreased hearing in
5 the right ear recently. Past medical history is significant for obesity, hyperlipidemia, seasonal allergies, and diet-controlled diabetes mellitus.
6 Otoscopy shows a small perforation in the right tympanic membrane and a pearty mass behind the membrane. Conduction hearing loss is noted
7 in the right ear. The remainder of the ear, nose, and throat examination is normal. Which of the following is the most likely cause of this patient's
8 aural mass?

9
0 A. Cholesterol and lipid accumulation

11 0 B. Facial nerve neuroma


12
13
0 C. Malignant squamous cell neoplasm

14 0 D. Noncaseating granuloma
15 0 E. Squamous cell debris
16
17
18
Submit
19
20
21
22
23
24

~ Ill ® 0
Block Time Rematntng: 00:00:58
TUTOR Feedback Suspend End Block

••
•• -·
~ ·
o· e mw []
t

• ~
~·-
I~ if A ~·
1LJS PM
t?i; ~·)) 2/12/2019 ~
X
1
2
=
-
Item 10 of 24
Questoon ld: 11628 Ia \='Mark
<J
Previous
[>
Next
?
Tut o ria l
4l
Lab Values
~
N ot es
~
Calculator
,
Reverse Color
aiiAJ 0
Text Zoom
3 •
4 A 48-year-old woman comes to the office due to an intermittent ear discharge over the last 2 years. She has also noticed decreased hearing in
5 the right ear recently. Past medical history is significant for obesity, hyperlipidemia, seasonal allergies, and diet-controlled diabetes mellitus.
6 Otoscopy shows a small perforation in the right tympanic membrane and a pearty mass behind the membrane. Conduction hearing loss is noted
7 in the right ear. The remainder of the ear, nose, and throat examination is normal. Which of the following is the most likely cause of this patient's
8 aural mass?
9
A. Cholesterol and lipid accumulation (39%]

11 B. Facial nerve neuroma [10%]


12
C. Malignant squamous cell neoplasm (6%]
13
14 D. Noncaseating granuloma (5%]
15 E. Squamous cell debris [37%]
16
17
18
Om1tted
19 llo. 37% fi\ 4 Seconds ~ 09/05/2018
Correct answer l!!.!.
20 Answered correcUy \.::J Time Spent EE3 Last Updated
E
21
22
23 Explanation
24
Cholesteatomas are collections of squamous cell debris that form a round, pearly mass behind the tympanic membrane in the m iddle ear. They
can occur congenitally or may develop in adults as either an acquired primary lesion or secondary to infection, trauma, or surgery of the middle
ear. Primary cholesteatomas are a result of chronic negative pressure in the middle ear causing retraction pockets in the tympanic membrane that
X
1
2
=
-
Item 10of24
Questoon ld: 11628 Ia \='Mark
<J
Previous
[>
Next
?
Tutorial
4l
Lab Values
~
Notes
~
Calculator
,
Reverse Color
aiiAJ 0
Text Zoom
3 Explanatoon
4
5 Cholesteatomas are collections of squamous cell debris that form a round, pearly mass behind the tympanic membrane in the middle ear. They
6 can occur congenitally or may develop in adults as either an acquired primary lesion or secondary to infection. trauma, or surgery of the middle
7 ear. Primary cholesteatomas are a result of chronic negative pressure in the middle ear causing retraction pockets in the tympanic membrane that
8 become cystic; as the squamous cell debris accumulates, a cholesteatoma is formed. Secondary cholesteatomas occur after squamous
9 epithelium migrates to or is implanted in the middle ear f'skin in the wrong place").

Cholesteatomas most commonly cause painless otorrhea. They also can produce lytic enzymes and are often discovered when they erode
11 through the auditory ossicles, causing conductive hearing loss. If a mass grows sufficiently large, it can erode into the vestibular apparatus or
12 facial nerve, causing vertigo or facial palsies.
13
(Choice A) Cholesterol granulomas can form in the middle ear after hemorrhage but are much less common than cholesteatomas. They appear
14
as bluish-black gelatinous material behind the tympanic membrane. Despite what the name implies, cholesteatomas do not contain any lipid or
15
cholesterol components.
16
17 (Choice B ) A facial nerve neuroma can grow in the middle ear as the facial nerve courses through this territory. However, it would present with
unilateral facial paralysis.
18
19 (Choice C) Squamous cell carcinoma is the most common malignant tumor of the ear canal. It typically appears as an ulcerated plaque or
20 nodule. The most common symptom is local or regional pain.
21
(Choi ce D) Granu lomatous disease of the ear can occur but is uncommon and usually develops in conjunction with a systemic disease, such as
22
sarcoidosis, granulomatosis with polyangiitis, or Langerhans cell histiocytosis.
23
24
Educational objective :
Cholesteatomas are collections of squamous cell debris that form a mass behind the tympanic membrane. Cholesteatomas can be congenital or
may occur as an acquired primary lesion or following infection, trauma, or surgery of the middle ear. They can cause hearing loss due to erosion
into auditory ossicles.

~ Ill ® 0
Block Time Rematntng: 00:01:00
TUTOR Feedback Suspend End Block

••
•• -·
~ ·
o· []
t

• ~
~--
I~ if A

1LJS PM
t?i; ~·)) 2/12/2019 ~
X
1
= ltem10of24 = <] C> ? 4f ~ ~ , ~ 0
2 - Qu estion ld: 11628 ,. ? Mark Previous Next Tut orial Lab Values Notes Calculator Reverse Color Text zoom
3 Explanation
4 Exhibit Display dl ~
5
Middle ear anatomy
6
7 Temporal bone
8
9
• 10 I
11 Tensor tympani muscle
12
13
14
15
16
~--- oval
17 window
18
19
Round
window
20
21
Tympanic
22 membrane
AUditOry
23
tube
cavity Stapedius
24
muscle

Zoom In Zoom Out Reset Add To Flash Card

~ IJI ® 0
Block Time Remaining : 00:01 :oo
TUTOR Feedback Suspend End Block

•• -o·
_,· lfi w -• rGJ rc:"~
11 :35 PM
•• IJ9 •• >f A ~? x ~
ft?. <l>l) 2/12/2019 ~
X
1
2
=
-
Item 11 of24
QuestiOn ld: n21 a'? Mark
<J
Previous
[>
Next
?
Tutorial
4l
Lab Values
~
Notes
~
Calculator
,
Reverse Color
aiiAJ
Text zoom
0
3
4 A 67-year-old smoker comes to the office w ith a 2-week history of decreased vision in his right eye that he describes as "blurry" and "distorted."
5 The patient has been having vision problems over the past year, and these have made it more difficult to drive and require that he use a bright light
6 to read the newspaper. He has no history of diabetes mellitus or hypertension. The patient uses an albuterol inhaler for occasional wheezing and
7 shortness of breath. Ophthalmologic examination of the right eye shows a grayish discoloration of the macula with areas of adjacent hemorrhage.
8 Which of the following should be specifically targeted in treatment of this patient's condition?
9
10
0 A. CD20 lymphocyte glycoprotein

• 11~ 0 B. Epidermal growth factor receptor


12
13
0 C. lnter1eukin-2

14 0 D. Tumor necrosis factor-alpha


15 0 E. Vascular endothelial growth factor
16
17
18
Submit
19
20
21
22
23
24

~ Ill ® 0
Block Time Rematntng: 00:01:07
TUTOR Feedback Suspend End Block

••
•• -·
~ ·
o· []
t

• ~
~--
I~ if A

1LJS PM
t?i; ~·)) 2/12/2019 ~
X
1
2
=
-
Item 11 of24
QuestiOn ld: n21 a'? Mark
<J
Previous
[>
Next
?
Tutoria l
4l
Lab Values
~
Notes
~
Calculator
,
Reverse Color
aiiAJ
Text zoom
0
3 •
4 A 67-year-old smoker comes to the office with a 2-week history of decreased vision in his right eye that he describes as "blurry" and "distorted."
5 The patient has been having vision problems over the past year, and these have made it more difficult to drive and require that he use a bright light
6 to read the newspaper. He has no history of diabetes mellitus or hypertension. The patient uses an albuterol inhaler for occasional wheezing and
7 shortness of breath. Ophthalmologic examination of the right eye shows a grayish discoloration of the macula with areas of adjacent hemorrhage.
8 Which of the following should be specifically targeted in treatment of this patient's condition?
9
A. CD20 lymphocyte glycoprotein (1%]
10
• 11~ B. Epidermal growth factor receptor (6%]
12
C. lnter1eukin-2 (3%]
13
14 D. Tumor necrosis factor-alpha (6%]
15 E. Vascular endothelial growth factor (82%]
16
17
18
Omttted
19 fi\ 9 Seconds ~ 10/17/2018
Correct answer "''
l!!.!. 82%
20 Answered correcUy \.::J Time Spent EE3 Last Updated
E
21
22
23 Explanation
24
This patient likely has age-related macular degeneration (AMD), the leading cause of blindness in industrialized countries. AMD occurs in
genetically predisposed individuals as a result of advancing age and environmental factors (eg, smoking) and is classified into dry and wet
subtypes. Dry AMD is characterized by gradual vision loss in one or both eyes and can cause difficulty with driving/reading. The condition likely
X
1
2
=
-
Item 11 of24
QuestiOn ld: n21 a'?Mark
<J
Previous
[>
Next
?
Tutorial
4l
Lab Values
~
Notes
~
Calculator
,
Reverse Color
aiiAJ
Text zoom
0
. .
3
genetically predisposed individuals as a result of advancing age and environmental factors (eg, smoking) and is classified into dry and wet
4
subtypes. Dry AMD is characterized by gradual vision loss in one or both eyes and can cause difficulty with driving/reading. The condition likely
5
results from chronic oxidative damage to the retinal pigment epithelium and choriocapillaris, leading to subretinal inflammation with abnormal
6
extracellular matrix formation (eg, confluent drusen, basement membrane thickening). These changes appear on funduscopy as subretinal
7
drusen deposits with pigment abnormalities.
8
9 Progressive extracellular matrix accumulation can eventually result in retinal hypoxia, which stimulates local vascular endothelial growth factor

10
(VEGF) production and causes subretinal neovascularization with formation of leaky vessels. This condition is termed wet AMD and presents with
acute vision loss (days to weeks) with metamorphopsia (distortion of straight lines). Funduscopy shows a grayish-green subretinal discoloration
• 11~
with adjacent fluid/hemorrhage.
12
13 Patients with dry or wet AMO may benefit from antioxidant vitamins and zinc, and smokers should receive smoking cessation counseling to
14 prevent disease progression. Wet AMO usually requires specific treatment with VEGF inhibitors (eg , ranibizumab, bevacizumab).
15
(Choice A) Therapy against the C020 glycoprotein on B cells (eg, the monoclonal antibody rituximab) is used to treat various lymphomas and
16 active rheumatoid arthritis.
17
{Choice B) Epidermal growth factor receptor inhibitors (eg, erlotinib, gefitinib) are used to treat advanced non-small cell lung cancer.
18
19 {Choice C) Anti.interleukin-2 therapy is primarily used for immunosuppression in organ transplant patients and in conditions such as graft versus
20 host disease.
21
{Choice D) Inhibitors of tumor necrosis factor-alpha are used in many inflammatory autoimmune conditions, such as rheumatoid arthritis,
22 inflammatory bowel disease, and seronegative spondyloarthropathies.
23
Educational objective:
24
Wet age-related macular degeneration is characterized by retinal neovascularization due to increased vascular endothelial growth factor (VEGF)
levels. Patients typically have acute vision loss and metamorphopsia with funduscopy showing a grayish-green subretinal membrane and/or
subretinal hemorrhage. Treatment includes smoking cessation and VEGF inhibitor therapy (eg, ranibizumab, bevacizumab).

~ Ill ® 0
Block Time Rematntng: 00:01:09
TUTOR Feedback Suspend End Block

••
•• -·
~ ·
o· []
t

• ~
~--
I~ if- A

11:36 PM
t?i; ~·)) 2/12/20 19 ~
X
1
= Item 11 of 24 = <] C> ? 4f ~ ~ , ~ 0
2 - Question ld: IT21
.
,. ? Mark Previous
. Next Tut orial Lab Values Notes Calculator Reverse Color Text zoom
3
~ - .. - ~ ~a A a ~ ~ ~ .. A - .. A • .. A - - A a 4 - • •• .. a 4 .. • a 4 - • - a a 4 - - • • .. A

4 Exhibit Display dl ~
5 AMD , Calcified Drusen (#1) AMD, Exudative
6
7
8
9
10

12
13
14
15
16
17
18
19
20
21
22
23
24

Zoom In Zoom Out Reset Add To Flash Card

~ IJI ® 0
Block Time Remaining: 00:01:09
TUTOR Feedback Suspend End Block

•• -·
_, lfi w -• rGJ rc:"~
11 :36 PM
•• o· IJ9 •• >f A ~? x ~
ft?. <l>l) 2/12/2019 ~
X
1
= Item 11 of 24 = <] C> ? 4f ~ ~ , ~ 0
2 - Question ld: IT21
.
,. ? Mark Previous
. Next Tut orial Lab Values Notes Calculator Reverse Color Text zoom
3
~ - .. - ~ ~a A a ~ ~ ~ .. A - .. A • .. A - - A a 4 - • •• .. a 4 .. • a 4 - • - a a 4 - - • • .. A

4 Exhibit Display dl ~
5
6
7
8
9
10

12
13
14
15
16
17
18
19
20
21
22
23
24

Zoom In Zoom Out Reset


X
1
= Item 11 of 24 = <] C> ? 4f ~ ~ , ~ 0
2 - Qu estion ld : IT21
.
,. ? Mark Previous
. Next Tut orial Lab Values Notes Calculator Reverse Color Text zoom
3
4
5
6
7
8
9
10

12
13
14
15
16
17
18
19
20
21
22
23
24

Zoom In Zoom Out Reset Add To Flash Card

~ IJI ® 0
Block Time Remaining : 00:01 :09
TUTOR Feedback Su spend End Block

•• -o·
_,· lfi w -• rGJ rc:"~
11 :36 PM
•• IJ9 •• >f A ~? x ~
ft?. <l>l) 2/12/2019 ~
X
1
2
=
-
Item 12of 24
Questoon ld: 1363 Ia \='Mark
<J
Previous
[>
Next
?
Tutorial
4l
Lab Values
~
Notes
~
Calculator
,
Reverse Color
aiiAJ
Text Zoom
0
3
4 A 64-year-old African American woman is brought to the clinic by her husband due to a fall. She was walking in her living room when she tripped
5 over an electrical cord. The patient has had several other ground-level falls in the last 6 months but has had no significant injuries. She also gave
6 up driving a year ago follov'ling a near-miss motor vehid e accident Past medical history is notable for hypertension and type 2 diabetes mellitus,
7 for which she takes appropriate medications. Mental status and neurologic examinations are normal. Ophthalmic examination reveals reduced
8 vision in her peripheral visual fields bilaterally and an elevated intraocular pressure. After discussion of treatment options, the patient is initiated
9 on timolol ophthalmic drops. Which of the following structures is the most likely target of this medication?

10
11
O A. Ciliary epithelium

0 B. Ciliary muscle
13
14
0 C . lens

15 0 D. Pupil sphincter

16
17
0 E. Trabecular meshwork

18
19
Submit
20
21
22
23
24

~ Ill ® 0
Block Time Rematntng: 00:01:10
TUTOR Feedback Suspend End Block

••
•• -·
~ ·
o· []
t

• ~
~·-
I~ if A

11:36 PM
t?i; ~·)) 2/12/2019 ~
X
1
2
= Item 12of24 <J [> ? &I ~ ~ , ail!) 0
- Questoon ld: 1363 Ia \='Mark Previous Next Tut o ria l Lab Values Notes Calculator Reverse Color Text Zoom
3 •
4 A 64-year-old African American woman is brought to the clinic by her husband due to a fall. She was walking in her living room when she tripped
5 over an electrical cord. The patient has had several other ground-level falls in the last 6 months but has had no significant injuries. She also gave
6 up driving a year ago follov'ling a near-miss motor vehid e accident Past medical history is notable for hypertension and type 2 diabetes mellitus,
7 for which she takes appropriate medications. Mental status and neurologic examinations are normal. Ophthalmic examination reveals reduced
8 vision in her peripheral visual fields bilaterally and an elevated intraocular pressure. After discussion of treatment options, the patient is initiated
9 on timolol ophthalmic drops. Which of the following structures is the most likely target of this medication?

10
A. Ciliary epithelium [42%)
11
B. Ciliary muscle [27%)
13
c. Lens [0%)
14
15 D. Pupil sphincter [7%1
16 E. Trabecular meshwork [20%)
17
18
19
20 Om1tted llo. 42% f'i"' 3 Seconds F!;:¢1 10/07/2018
Correct answer l!.!.!.
21
Answered correcUy '\.::.) Time Spent EE3 Last Updated
A
22
23
24 Explanation

Glaucoma medications
X
1
2
= Item 12of 24 <J [> ? &I ~ ~ , ail!) 0
- Questoon ld: 1363 Ia \='Mark Previous Next Tutorial Lab Values Notes Calculator Reverse Color Text zoom
3
4 Glaucoma medications
5
6
7
Trabecular
8
9
10
11

13
14
Trabec ular outflow
15
• Muscarinic agonists
16
17
18 Schlemm
19 canal
20
21 Aqueous humor inflow
22
• Beta blockers
23 Uveoscleral outflow • Alpha-2 AR agonists
24 • Prostaglandin agonists • CAis

Alpha-2 AR agonists =alpha-2 adrenergic receptor agonists; CAis =carbonic anhydrase inhibitors.
"

~ Ill ® 0
Block Time Rematntng: 00:01:12
TUTOR Feedback Suspend End Block

••
•• _,
o• []
t

• ~
~··

·~ If A

11:36 PM
!I;; ~·)) 2/12/2019 ~
1
2
3
4 Aqueous humor inflow
5 • Beta blockers
6 Uveoscleral outflow • Alpha-2 AR agonists
7 • Prostaglandin agonists • CAis
8
9
Alpha-2 AR agonists = alpha-2 adrenergic receptor agonists; CAis =carbonic anhydrase inhibitors.
10
C>UWOI1d
11
This patient with chronic, progressive loss of peripheral vision has typical features of open-angle glaucoma. Glaucoma is a type of optic
neuropathy characterized by atrophy of the optic nerve head. It is more common with advancing age and is especially common in certain
13
ethnicities (eg, African Americans). Glaucoma is usually associated with elevated intraocular p ressure (lOP) due to increased production or
14
decreased outflow of aqueous humor. Aqueous humor is produced by the epithelial cells of the ciliary body. It is secreted into the posterior eye
15
chamber and transferred through the pupil into the anterior eye chamber. The anterior chamber angle (iridocorneal angle) contains a trabecular
16
meshwork through which the aqueous humor diffuses into Schlemm's canal (scleral venous sinus) and subsequently into episcleral and
17
conjunctival veins.
18
19
Diagnostic features of glaucoma include elevated lOP and abnormal visual field testing with decreased peripheral vision . Funduscopic
examination will show an increased cup-to-disc ratio due to loss of ganglion cell axons. The goal of treatment in glaucoma is to decrease lOP
20
w ith drugs that either decrease the production of aqueous humor or increase its outflow. limolol and other nonselective beta blockers work by
21
diminishing the secretion of aqueous humor by the ci liary epithelium.
22
23 (Choices B , C, D, and E) The intraocular muscles of the eye, incl uding the ciliary muscle and pupillary sphincter of the iris, receive
24 parasympathetic innervation from cholinergic fibers of the oculomotor nerve (cranial nerve Ill). Cholinergic agonists (eg, pilocarpine) cause miosis
by promoting contraction of the sphincter of the iris. This causes the anterior chamber angle to become wider and makes the trabecular
meshwork more accessible to outflow of aqueous humor. Cholinergic agonists also promote ciliary muscle contraction, causing the lens to
become more convex.

~ Ill ® 0
Block Time Remammg: 00:01:12
TUTOR Feedback Suspend End Block

••
•• o_,·
~ ·

[]
t

• ~
~--

·~ If A ~· ri?. ~·))
11 :36 PM
2/12/Z0 19 ~
X
1
2
= Item 12of24 <J [> ? 4l ~ ~ , ail!) 0
- Questoon ld: 1363 Ia \='Mark Previous Next Tutorial Lab Values Notes Calculator Reverse Color Text Zoom
3
. ... ·- ... ·····------- --·- . ...... ··-- .. .
CDUWOI1d
4
This patient with chronic, progressive loss of peripheral vision has typical features of open-angle glaucoma. Glaucoma is a type of optic
5
neuropathy characterized by atrophy of the optic nerve head. It is more common with advancing age and is especially common in certain
6
ethnicities (eg, African Americans). Glaucoma is usually associated with elevated intraocular pressure (lOP) due to increased production or
7
decreased outflow of aqueous humor. Aqueous humor is produced by the epithelial cells of the ciliary body. It is secreted into the posterior eye
8
chamber and transferred through the pupil into the anterior eye chamber. The anterior chamber angle ~ridocomeal angle) contains a trabecular
9
meshwork through which the aqueous humor diffuses into Schlemm's canal (scleral venous sinus) and subsequently into episcleral and
10
conjunctival veins.
11
Diagnostic features of glaucoma include elevated lOP and abnormal visual field testing with decreased peripheral vision. Funduscopic
13 examination will show an increased cup-to-disc ratio due to loss of ganglion cell axons. The goal of treatment in glaucoma is to decrease lOP
14
with drugs that either decrease the production of aqueous humor or increase its outflow. Timolol and other nonselective beta blockers work by
diminishing the secretion of aqueous humor by the ciliary epithelium.
15
16 (Choices B, C, D, and E) The intraocular muscles of the eye, including the ciliary muscle and pupillary sphincter of the iris, receive
17 parasympathetic innervation from cholinergic fibers of the oculomotor nerve (cranial nerve Ill). Cholinergic agonists (eg, pilocarpine) cause miosis
18 by promoting contraction of the sphincter of the iris. This causes the anterior chamber angle to become wider and makes the trabecular
19 meshwork more accessible to outflow of aqueous humor. Cholinergic agonists also promote ciliary muscle contraction, causing the lens to
20 become more convex.
21 Educo:ltional objective :
22 Open-angle glaucoma is characterized by progressive loss of peripheral vision from elevated intraocular pressure. Timolol and other nonselective
23 beta blockers work by diminishing the secretion of aqueous humor by the ciliary epithelium. Acetazolamide, a carbonic anhydrase inhibitor, also
24 decreases aqueous humor secretion by the ciliary epithelium. Prostaglandin F2a (eg, latanoprost, travoprost) and cholinomimetics (eg,
pilocarpine, carbachol) decrease intraocular pressure by increasing the outflow of aqueous humor.

Copyright® UWOI1d All ngllts reserved


1
2
3
4
5 Glaucoma pathophysiology
6
Nonnal eye
7
8
9
10
11

13
14
Nonnal fluid flow
15
16 Glaucoma
17
18
19
Elevated
20 pre1-sure

21
22
23
OptiC nerve
24 damage B loc~ed fluid flow

Zoom In Zoom Out Reset Add To Flash Card


1
2
3
4
5 Optic disc in glaucoma
6
7 Normal Open-angle glaucoma
8
9
10
11

13
14 Disc Cup
15
16
17
18
19
20
21 o Enlarged cup with cup:disc ratio >0.6
22 o Cup:disc ratio <0.5 o Increase in cup size over time
23 o Clear disc rim o Thinning of disc rim
24 o Pale disc (optic nerve atrophy)

Zoom In Zoom Out Reset A dd To Flash Card


X
1
2
=
-
Item 13of24
QuestiOn ld: 1195 a'? Mark
<J
Previous
[>
Next
?
Tut o ria l
&I
Lab Values
~
Notes
~
Calculator
,
Reverse Color
aiiAJ 0
Text zoom
3 •
4 A 6-year-old asthmatic has the finding shown in the image below on routine examination.
5
6
7
8
9
10
11
12

14
15
16
17
18
19
20
21
22
23
24
1
2
3
4
5
6
7
8
9
10
11
12

14 Which of the following is the most appropriate pharmacotherapy for the lesions on this patient's oral mucosa?
15
16 O A. Acyclovir
17
18
0 B. Amphotericin-B

19 0 C. Griseofulvin
20
21
0 D. Nystatin

22 0 E. Penicillin

23 0 F. Terbinafine
24

Submit
1
2
3
4
5
6
7
8
9
10
11
12

14 Which of the following is the most appropriate pharmacotherapy for the lesions on this patient's oral mucosa?
15
16 A. Acyclovir [1%]
17
B. Amphotericin-B (12%)
18
19
C. Griseofulvin [4%]
20 D. Nystatin [77%]
21
E. Penicillin (0%)
22
23 F. Terbinafine [2%]
24

Omitted
11 /24/2018
X
1
2
=
-
Item 13of24
Questoon ld: 1195 Ia \='Mark
<J
Previous
[>
Next
?
Tut o ria l
4l
Lab Values
~
Notes
~
Calculator
,
Reverse Color
aiiAJ
Text Zoom
0
3
4 The thick white patches seen on this patient's tong ue are indicative of oral thrush (oropharyngeal candidiasis). Thrush is a common variant of
5 localized infection with Candida fungi. Oral thrush is common in young infants due to immaturity of the newborn immune system. In older children
6 and adults, it is associated with immunosuppression and frequently occurs in patients with HIV infection, diabetes mellitus, and cancer. Asthma
7 patients who take oral or inhaled steroids are also at risk for candidiasis. Systemic antibiotic therapy and dentures are other predisposing factors.
8 In immunocompetent patients, localized candidiasis is treated with topical antifungal medications. Nystatin is a polyene antifungal with a
9 mechanism of action similar to amphotericin B. It binds to ergosterol molecules in the fungal cell membrane, causing pores and leakage of fungal
10 cell contents. Nystatin is not absorbed from the gastrointestinal tract. It is administered as an oral agent ['swish and swallow") for oropharyngeal
11 candidiasis.
12
(Choice A) Acyclovir is an antiviral drug effective against herpes simplex virus (1 and 2) and varicella zoster virus. It is a nucleoside analog that
is converted into acyclo-GTP in infected cells, which inhibits viral DNA polymerase.
14
15 (Choice B ) Amphotericin B is a polyene antifungal similar to nystatin used for systemic mycoses. It is administered intravenously and never used

16
for simple mucocutaneous infections due to many toxic side effects.

17 (Choice C) Griseofulvin inhibits fungal cell mitosis at metaphase. It is indicated for the treatment of dermatophytoses, not candidiasis.
18
(Choice E) Penicillins are j3-lactam antibiotics that inhibit the formation of peptidoglycan cross-links in bacterial cell walls.
19
20 (Choice F) Terbinafine is an allylamine antifungal that accumulates in the skin, nai ls, and adipose tissue. It is used to treat dermatophytosis
21 (onychomycoses), not cand idiasis.

22 Educational objective :
23 Nystatin is a polyene antifungal and the drug of choice for oropharyngeal candidiasis in patients without advanced immunodeficiency. It acts by
24 binding to ergosterol in the fungal cell membrane, causing the formation of pores and leakage of fungal cell contents. Nystatin is not absorbed
from the gastrointestinal tract and is administered as an oral "swish and swallow" agent.

Copyright® UWOI1d All ngllts reserved


~ OJ X
1
=
-
ltem14of24
Q t ld·. '~o R DMark
<J
.
[> ? 4l ~ ~ , ~
~
0
2 ues 1on .J'U f Prev1ous Next Tutoria l Lab Values Notes Calculator Reverse Color Text Zoom
3 •
4 A 64-year-old man comes to the emergency department due to painless loss of vision in the right eye. The patient's symptoms started suddenly
5 several hours ago and have persisted. He has had no trauma or any similar episode in the past. He has a history of coronary artery disease,
6 atrial fibrillation, and type 2 diabetes mellitus. Blood pressure is 144/86 mm Hg and pulse is 92/min. Cardiovascular examination reveals
7 irregularly irregular rhythm with no murmurs, no peripheral edema, and slightly diminished pedal pulses in both feet symmetrically. Visual acuity in
8 the right eye is hand motion only and normal in the left eye. Funduscopic examination of the right eye is shown in the image below.
9
10
11
12
13

15
16
17
18
19
20
21
22
23
24
~
1
2
3
4
5
6
7
8
9
10
11
12
13

15
16
17
18
19
20
21
22
23
24

Which of the following is the most likely cause of this patient's loss of vision?
~
1
2
3
4
5
6
7
8
9
10
11
12
13

15
Which of the following is the most likely cause of this patient's loss of vision?
16
17
18
O A. Amaurosis fugax

19 0 B. Diabetic retinopathy

20
0 c. Pituitary adenoma
21
22 0 D. Retinal artery occl usion

23 0 E. Temporal lobe stroke


24

Submit
~
1
2
3
4
5
6
7
8
9
10
11
12
13

15
Which of the following is the most likely cause of this patient's loss of vision?
16
17
A. Amaurosis fugax (7%]
18
19
B. Diabetic retinopathy (14%]

20 c. Pituitary adenoma (0%]


21
D. Retinal artery occl usion (76%]
22
23 E. Temporal lobe stroke (1%]
24

Omitted
1
2
3
4
5 retinal artery occlusion
6
7
8
9
10
11

12
13

15
16
17
Cherry red
18
19
macula I
Retinal whitening

20
21
22
23
24
~
1
2
3
4
5 Acute and painless monocular vision loss is characteristic of central reti nal artery occlusion (CRAO). The vision loss includes the entire visual
6 field and is often permanent. Funduscopic findings include a pale retina (due to ischemia and edema) and a cherry-red macula (the fovea and
7 foveola are thin and have a separate blood supply from the choroid artery; the red choroid is easily visible underneath).
8
The central retinal artery is a branch of the ophthalmic artery, which arises from the internal carotid artery. Athero- and thromboembolism are
9
the most common causes of CRAO. Predisposing conditions include atrial fibrillation and carotid artery stenosis. CRAO may also be caused by
10
vasculitic diseases (eg giant cell arteritis).
11
12 (Choice A) Amaurosis fugax is a painless, transient, monocular vision loss caused by a small embolus to the ophthalmic artery. It usually does
not last more than a few seconds.
13
(Choice B ) Diabetic retinopathy manifests with blurry vision, black spots, floaters, and decreased peripheral vision. Acute visual loss may occur
15 in the setting of a complication such as vitreous hemorrhage, but this patient does not have evidence of neovascularization or hemorrhage.
16
(Choice C) Large pituitary adenomas may compress the central part of the optic chiasm, causing bitemporal hemianopia.
17
18 (Choice E) The lower division of the geniculocalcarine tract (Meyer loop), which carries impulses from the lower retina, passes through the

19 temporal lobe. Temporal lobe infarcts involving the Meyer loop cause contralateral upper quadrantanopsia ("pie in the sky" visual defect).

20 Educational objective :
21 Central retinal artery occlusion presents with sudden, pain less, and permanent monocular blindness. Funduscopic examination reveals a pale
22 retina and a "cherry-red" macula.
23
24
References

• A review of central retinal artery occlusion: clinical presentation and management

Copyright ® UWor1d AI nghls tese<Ved

~ Ill ® 0
Block Time Remammg: 00:01:31
TUTOR Feedback Suspend End Block

••
•• -·
~ ·
o· []
t

• ~
~--
I~ if- A

11:37PM
t?i; ~·)) 2/12/20 19 ~
~ OJ X
1
2
=
-
Item 15of 24
Questoon ld: 1345 Ia \='Mark
<J
Previous
[>
Next
?
Tutorial
4l
Lab Values
~
Notes
~
Calculator
,
Reverse Color
aiiAJ
Text Zoom
0
3
4 A 35-year-old man comes to the physician because of a 2-week history of nasal congestion. He has used a topical decongestant every few hours
5 since his symptoms began. He experienced relief for almost 1 week, but then his nasal congestion returned . The patient has a history of allergic
6 rhinitis and has had episodes of rhinorrhea in the past, but none of them lasted longer than a few days. He denies fever, throat pain , headaches,
7 cough, and lymph node enlargement. Aside from his allergic rhinitis, the patient has no other medical problems. Physical examination shows
8 nasal mucosa that appears edematous and red with a few areas of punctate bleeding. The remainder of the examination reveals no
9 abnormalities. Which of the following is the most appropriate next step in the management of this patient?

10
11
O A. Stop the decongestant

12 0 B. SWitch to ephedrine
13
14
0 C. Add oral corticosteroids

0 D. Add antihistamines

16
17
0 E. Start antibiotics

18
19
Submit
20
21
22
23
24

~ Ill ® 0
Block Time Rematntng: 00:01:33
TUTOR Feedback Suspend End Block

••
•• -·
~ ·
o· e mw []
t

• ~
~·-
I~ if A ~·
11:38 PM
t?i; ~·)) 2/12/2019 ~
~ OJ X
1
2
=
-
Item 15of24
Questoon ld: 1345 Ia \='Mark
<J
Previous
[>
Next
?
Tut o ria l
4l
Lab Values
~
Notes
~
Calculator
,
Reverse Color
aiiAJ
Text Zoom
0
3 •
4 A 35-year-old man comes to the physician because of a 2-week history of nasal congestion. He has used a topical decongestant every few hours
5 since his symptoms began. He experienced relief for almost 1 week, but then his nasal congestion returned . The patient has a history of allergic
6 rhinitis and has had episodes of rhinorrhea in the past, but none of them lasted longer than a few days. He denies fever, throat pain , headaches,
7 cough, and lymph node enlargement. Aside from his allergic rhinitis, the patient has no other medical problems. Physical examination shows
8 nasal mucosa that appears edematous and red with a few areas of punctate bleeding. The remainder of the examination reveals no
9 abnormalities. Which of the following is the most appropriate next step in the management of this patient?

10
A. Stop the decongestant (67%]
11
12 B. Switch to ephedrine (8%]
13
c. Add oral corticosteroids (5%]
14
D. Add antihistamines (1 5% ]
16 E. Start antibiotics [2%]
17
18
19
Om1tted
20
Correct answer
llo. 67% (1\ 4 Seconds F!r¢:1 11 /29/2018
21
l!.!.!. Answered correcUy \.:::.) Time Spent EE3 Last Updated
A
22
23
24 Explanation

Vasoconstriction by a-adrenergic agonists is prominent in the vessels of the nasal mucosa, making these medications effective decongestants.
Phenylephrine, xylometazoline, and oxymetazoline are used as topical preparations for the treatment of allergic rhinitis and common cold
~
1
2
3
4
5 Vasoconstriction by a-adrenergic agonists is prominent in the vessels of the nasal mucosa, making these medications effective decongestants.
6 Phenylephrine, xylometazoline, and oxymetazoline are used as topical preparations for the treatment of allergic rhinitis and common cold
7 associated congestion and rhinitis.
8 These medications, however, are characterized by rapidly dedining effect after a few days of use. This phenomenon is called tachyphylaxis. It
9 occurs because of decreased production of endogenous norepinephrine from the nerve tenninals due to a negative feedback mechanism,
10 resulting in relative vasodilation (ie, removal of normal vasoconstrictive tone) and subsequent edema and congestion. This leads to exacerbation
11 of the nasal congestion symptoms. Rebound rhinorrhea (ie, rhinitis medicamentosa) is described as nasal congestion without cough, sneezing, or
12 postnasal drip. Rhinorrhea may or may not be present (note the paradox in naming). Rebound rhinorrhea is associated with the use of topical
13 decongestants for !! 3 days. The use of adrenergic agonists should be stopped to allow the restoration of normal norepinephrine feedback
14 pathways.

Another medication associated with the phenomenon of tachyphylaxis is nitroglycerine. The decrease in its effect is explained by the diminished
16 release of NO from the target cells. Drug-free intervals of 8-10 hours should be maintained during the use of nitroglycerine to prevent
17 tachyphylaxis.
18
(Choice B) Ephedrine is an indirect-acting adrenergic agonist used as a decongestant. Like the other adrenergic medications, it causes
19
tachyphylaxis after a few days of treatment. A switch to ephedrine would not improve this patient's rebound rhinorrhea.
20
21 (Choice C) This patient has been excessively using a decongestant; if he discontinues it and experiences excessive withdrawal symptoms, a
22 short-term course of topical corticosteroids can be considered for symptom control. However, oral corticosteroids would not be indicated.
23 (Choice D) Antihistamines are used for the treatment of allergic rhinitis, but they are not effective in treating rebou nd rhinorrhea.
24
(Choice E) Antibiotics are effective in the treatment of certain bacterial infections. They are not used in the treatment of rebound rhinorrhea.

Educational objective:
Topical preparations of a-adrenergic agonists cause vasoconstriction of the nasal mucosa vessels and are used as decongestants. Overuse of

~ Ill ® 0
Block Time Rematntng: 00:01 :35
TUTOR Feedback Suspend End Block

••
•• -·
~ ·
o· []
t

• ~
~--
I~ if A

11:38 PM
t?i; ~·)) 2/12/20 19 ~
~
1
2
3
4 of the nasal congestion symptoms. Rebound rhinorrhea (ie, rhinitis medicamentosa) is described as nasal congestion without cough, sneezing, or
5 postnasal drip. Rhinorrhea may or may not be present (note the paradox in naming). Rebound rhinorrhea is associated with the use of topical
6 decongestants for 2:. 3 days. The use of adrenergic agonists should be stopped to allow the restoration of normal norepinephrine feedback
7 pathways.

8 Another medication associated with the phenomenon of tachyphylaxis is nitroglycerine. The decrease in its effect is explained by the diminished
9 release of NO from the target cells. Drug-free intervals of 8-10 hours should be maintained during the use of nitroglycerine to prevent
10 tachyphylaxis.
11
(Choice B) Ephedrine is an indirect-acting adrenergic agonist used as a decongestant Like the other adrenergic medications, it causes
12
tachyphylaxis after a few days of treatment A switch to ephedrine would not improve this patient's rebound rhinorrhea.
13
14 (Choice C) This patient has been excessively using a decongestant; if he discontinues it and experiences excessive withdrawal symptoms, a
short-tenn course of topical corticosteroids can be considered for symptom control. However, oral corticosteroids would not be indicated.

16 (Choice D) Antihistamines are used for the treatment of allergic rhinitis, but they are not effective in treating rebound rhinorrhea.
17
(Choice E) Antibiotics are effective in the treatment of certain bacterial infections. They are not used in the treatment of rebound rhinorrhea.
18
19 Educatio nal objective :
20 Topical preparations of a-adrenergic agonists cause vasoconstriction of the nasal mucosa vessels and are used as decongestants. Overuse of
21 these drugs causes negative feedback, resulting in decreased norepineph rine synthesis and release from nerve endings, which diminishes their
22 effect (ie, tachyphylaxis).

23
24 References

• Rh1mbs medicamentosa

• Diagnos1ng rhm111s allerg1c vs. nonallergic.

~ Ill ® 0
Block Time Rematntng: 00:01 :35
TUTOR Feedback Suspend End Block

••
•• -·
~ ·
o· []
t

• ~
~--
I~ if A

11:38 PM
t?i; ~·)) 2/12/20 19 ~
~ OJ X
1
=
-
ltem16of 24
Q ld·•na R DMark
<J
.
[> ? 4l ~ ~ , ~
~
0
2 ues11on . .I'U f Prev1ous Next Tutorial Lab Values Notes Calculator Reverse Color Text Zoom
3
4 A 42-year-old woman comes to the office to discuss dizziness. She has a 6-month history of episodic bouts of a "'spinning sensation" associated
5 with nausea and ringing in her left ear. The patient also has difficulty hearing while holding her phone to the left ear, although hearing in her right
6 ear is normal. The dizziness improves spontaneously and she feels "fine" between episodes. Past medical history is notable for hypothyroidism,
7 for which she is on thyroid replacement therapy, and endometriosis, which led to a hysterectomy at age 38. The patient works as a supermarket
8 manager. She does not use tobacco or drink alcohol. The patient's condition is most likely the result of which of the following?
9
10
0 A. Demyelinating process in the central nervous system

11 0 B. Increased volume of endolymph in the inner ear


12
13
0 C. Inflammation of the vestibular labyrinth

14 0 D. Mass lesion at the cerebellopontine angle


• 15 0 E. Sclerosis of the ossicles

17
18
Submit
19
20
21
22
23
24

~ Ill ® 0
Block Time Rematntng: 00:01:36
TUTOR Feedback Suspend End Block

••
•• -·
~ ·
o· []
t

• ~
~·-
I~ if A

11:38 PM
t?i; ~·)) 2/12/20 19 ~
~ OJ X
1
=
-
ltem16of24
Q ld· • na R DMark
<J
.
[> ? &I ~ ~ , ~
~
0
2 ues11on . .I'U f Prev1ous Next Tutoria l Lab Values Notes Calculator Reverse Color Text Zoom
3 •
4 A 42-year-old woman comes to the office to discuss dizziness. She has a 6-month history of episodic bouts of a "'spinning sensation" associated
5 with nausea and ringing in her left ear. The patient also has difficulty hearing while holding her phone to the left ear, although hearing in her right
6 ear is normal. The dizziness improves spontaneously and she feels "fine" between episodes. Past medical history is notable for hypothyroidism,
7 for which she is on thyroid replacement therapy, and endometriosis, which led to a hysterectomy at age 38. The patient works as a supermarket
8 manager. She does not use tobacco or drink alcohol. The patient's condition is most likely the result of which of the following?
9
A. Demyelinating process in the central nervous system (6%]
10
11 B. Increased volume of endolymph in the inner ear [54%]
12
C. Inflammation of the vestibular labyrinth [18%]
13
14 D. Mass lesion at the cerebellopontine angle [16%]
• 15 E. Sclerosis of the ossicles [4%]

17
18
Omttted
19 llo. 54% fi\ 3 Seconds ~ 10/30/2018
Correct answer l!!.!.
20 Answered correcUy \.::J Time Spent EE3 Last Updated
B
21
22
23 Explanation
24

Common causes of vertigo


~ OJ X
1
=
-
ltem16of24
Q ld· • na RDMark
<J
.
[> ? &I ~ ~ , ~
~
0
2 ues11on . .I'U f Prev1ous Next Tut o ria l Lab Values Notes Calculator Reverse Color Text Zoom
3
4
Common causes of vertigo
5
6
Cause Features
7
8 • Recurrent vertigo
9 Mlmil~re • Increased pressure & • Ear fullness/pain
d isease volume of endolymph
10 • Unilateral hearing loss & tinnitus
11
12 • Brief episodes brought on by
BPPV • Otoliths in head movement
13
semicircular canals
14 • No auditory symptoms
• 15
Vestibular • Inflammation of • Single episode that can last days
17 neuritis vestibular nerve
• Severe vertigo but no hearing loss
(labyrinthitis) (viral or postviral)
18
19 BPPV = benign paroxysmal positional vertigo.
20 ~UWorld
21
This patient has Meniere disease, a disorder of the in ner ear characterized by increased volume and pressure of endolymph (en dolymphati c
22
hydrops) that is thought to be due to defective resorption of endolymph. The resultant distension of the endolymphatic system causes damage to
23
the vestibular and cochlear components of the inner ear. Meniere disease is characterized by the following triad:
24
1. low-frequency tinnitus , or ringing, in the affected ear, often accompanied by a feeling of fullness
2. Vertigo, the subjective sensation of spinning or motion in the absence of actual motion; commonly associated with lightheadedness,
nausea, and vomiting
~
1
2
3
4 This patient has Menh!re disease, a disorder of the inner ear characterized by increased volume and pressure of endolymph (endolymphatic
5 hydrops) that is thought to be due to defective resorption of endolymph. The resultant distension of the endolymphatic system causes damage to
6 the vestibular and cochlear components of the inner ear. Meniere disease is characterized by the following triad :
7
1. Low-frequency tinnitus, or ringing, in the affected ear, often accompanied by a feeling of fullness
8
2. Vertigo, the subjective sensation of spinning or motion in the absence of actual motion; commonly associated with lightheadedness,
9
nausea, and vomiting
10
3. Sensorineural hearing loss, which is variable in severity but usually worsens over time
11
12 (Choice A) Multiple sclerosis (MS) is characterized by patchy demyelination in the central nervous system that varies over time. The presentation
of MS is highly variable and may resemble Meniere disease, but most patients will have additional. extra-auditory symptoms (eg. visual symptoms,
13
sensory disruptions).
14
• 15 (Choice C) Labyrinthitis is inflammation of the vestibular nerve that causes acute-onset vertigo, nausea, and vomiting. It usually occurs in a
single episode following a viral syndrome.
17
(Choice D) A mass lesion at the cerebellopontine angle is most commonly an acoustic neuroma (schwannoma of CN VIII). An acoustic neuroma
18
could cause sensorineural hearing loss, vertigo, and tinnitus, but symptoms would be persistent and progressive rather than episodic.
19
(Choice E) Otosclerosis is an inherited condition seen in middle age. Patients present with conductive hearing loss due to bony overgrowth of
20
the footplate of the stapes. Vertigo does not occur.
21
22 Educational objective :
23 Meniere disease is characterized by tinnitus, vertigo, and sensorineural hearing loss. Its pathogenesis is related to an increased volume and
24 pressure of endolymph in the vestibular apparatus.

References
~ OJ X
1
2
=
-
Item 17of 24
Questoon ld: &631 Ia \='Mark
<J
Previous
[>
Next
?
Tutorial
4l
Lab Values
~
Notes
~
Calculator
,
Reverse Color
aiiAJ 0
Text Zoom
3
4 A 36-year-old man is rushed to the emergency department after the sudden onset of shortness of breath and difficulty swallowing. He has visited
5 the emergency department several times before for food and skin allergies. He is unconscious on arrival. Examination shows excessive
6 accessory respiratory muscle use and edematous swelling of his face, lips, and tongue. There is also scattered urticaria over his upper body.
7 Attempts at intubation are unsuccessful due to massive soft tissue edema involving his pharynx. A decision is made to perform an emergency
8 cricothyrotomy. The incision made during this procedure will most likely pass through which of the following structures?
9
10
0 A. Buccopharyngeal fascia and platysma

11 0 B. Platysma and thyroid isthmus


12
13
0 C. Pretracheal fascia and cricoid cartilage

14 0 D. Pretracheal fascia and prevertebral fascia


15 0 E. Superficial cervical fascia and cricothyroid membrane
16

18
Submit
19
20
21
22
23
24

~ Ill ® 0
Block Time Rematntng: 00:01:39
TUTOR Feedback Suspend End Block

••
•• -·
~ ·
o· []
t

• ~
~--
I~ if A

11:38 PM
t?i; ~·)) 2/12/2019 ~
~ OJ X
1
2
= Item 17of 24 <J [> ? &I ~ ~ , ail!) 0
- Questoon ld: &631 Ia \='Mark Previous Ne xt Tut o ria l Lab Values N ot es Calculator Reverse Color Text Zoom
3 •
4 A 36-year-old man is rushed to the emergency department after the sudden onset of shortness of breath and difficulty swallowing. He has visited
5 the emergency department several times before for food and skin allergies. He is unconscious on arrival. Examination shows excessive
6 accessory respiratory muscle use and edematous swelling of his face, lips, and tongue. There is also scattered urticaria over his upper body.
7 Attempts at intubation are unsuccessful due to massive soft tissue edema involving his pharynx. A decision is made to perform an emergency
8 cricothyrotomy. The incision made during this procedure will most likely pass through which of the following structures?
9
A. Buccopharyngeal fascia and platysma (1%)
10
11 B. Platysma and thyroid isthmus (4%)
12
C. Pretracheal fascia and cricoid cartilage (29%)
13
14 D. Pretracheal fascia and prevertebral fascia [1%)
15 E. Superficial cervical fascia and cricothyroid membrane [62%)
16

18
19 Om1tted llo. fi\ 3 Seconds
62% ~ 09/06/2018
Correct answer I.!!.!.
20 Answered correcUy \.::J Time Spent EE3 Last Updated
E
21
22
23 Explanation

24
Palpation of cricothyroid membrane

~ Ill ® 0
Block Time Rematntng: 00:01:4 1
TUTOR Feedback Suspend End Block

••
•• _,
o• []
t

• ~
~--

·~ If A ~x
11:38 PM
!I;; ~·)) 2/12/2019 ~
~ 01 X
1
2
= Item 17of 24 <J [> ? &I ~ ~ , ail!) 0
- Questoon ld: &631 Ia \='Mark Previous Next Tut o ria l Lab Values Notes Calculator Reverse Color Text zoom
3
4
Palpation of cricothyroid membrane
5
6
7
8 Laryngeal prominence
9
(Adam's apple)
10
11
12
13
14
15
16

18
19
20
21
22
23
24
Thyroid
cartilage
Cricothyroid
Trachea carti lage
~
1
2
3
4
5
6
7
8
9
10
11
12
13 Thyroid
14 cartilage
15 Trachea cartilage
16
gland Cricothyroid
membrane
18
19
20
Cricothyrotomy
21
22
23
24

~ Ill ® 0
Block Time Rematntng: 00:01:41
TUTOR Feedback Suspend End Block

••
•• _,
o• []
t

• ~
~--

·~ If A ~x
11:38 PM
!I;; ~·)) 2/12/2019 ~
~ OJ X
1
2
= Item 17of 24 <J [> ? &I ~ ~ , ail!) 0
- Questoon ld: &631 Ia \='Mark Previous Next Tuto ria l Lab Values Notes Calculator Reverse Color Text Zoom
3 Thyroid carti lage •
4
5
6 Cricothyroid
membrane
7
8
9 1 - - - - - -- T - - -<:ricoid cartilage
10
11
12
13
14 ~-------------Trachea
15
16

18
19
20
21
22 @UWorld
23
This patient is experiencing acute, life-threatening laryngeal edema as part of an anaphylactic reaction precipitated by exposure to an allergen.
24
Laryngeal edema can initially present with throat tightness, difficulty swallowing, dyspnea, and hoarseness. However, the condition can quickly
progress to compromise the airway and cause asphyxiation.

Cricothyrotomy is indicated when an emergency airway is required and orotracheal or nasotrachealmtubation is either unsuccessful or
~ OJ X
1
2
= Item 17of 24 <J [> ? &I ~ ~ , ail!) 0
- Questoon ld: &631 Ia \='Mark Previous Next Tutorial Lab Values Notes Calculator Reverse Color Text Zoom
3
4 © UWor1d
5
This patient is experiencing acute, life-threatening laryngeal edema as part of an anaphylactic reaction precipitated by exposure to an allergen .
6 Laryngeal edema can initially present with throat tightness, difficulty swallowing, dyspnea, and hoarseness. However, the condition can quickly
7 progress to compromise the airway and cause asphyxiation.
8
Cricothyrotomy is indicated when an emergency airway is required and orotracheal or nasotracheal intubation is either unsuccessful or
9
contraindicated (eg, due to massive hemorrhage, vomiting, facial trauma, or airway obstruction). The procedure establishes an airway through the
10
placement of a tube between the cricoid and thyroid cartilages and requires an incision though the following structures:
11
12 1. Skin
13 2. Superficial cervical fascia Onduding subcutaneous fat and platysma muscle)
14 3. Investing and pretracheal layers of the deep cervical fascia

15 4. Cricothyroid membrane

16 (Choice A) The buccopharyngeal fascia extends from the carotid sheath to invest the pharyngeal constrictor muscles. This fascial layer lies
anterior to the prevertebral fascia, forming the retropharyngeal space between them . In fections involving the retropharyngeal space can extend
18 directly into the superior mediastinum and cause acute necrotizing mediastinitis.
19
(Choices B and C) The cricoid cartilage lies inferior to the thyroid cartilage at the level of C6 vertebra and the thyroid isthmus lies inferior to the
20
cricoid carti lage. The cricothyrotomy incision does not pierce these structures.
21
22 (Choice D) The deep cervical fascia is composed of three layers: investing, pretracheal, and prevertebral. The prevertebral layer ofthe deep

23 fascia is not penetrated during cricothyrotomy.

24 Educational object ive :


Cricothyrotomy is indicated when an emergency airway is required and orotracheal or nasotracheal intubation is either unsuccessful or
contraindicated. The cricothyrotomy incision passes through the superficial cervical fascia, pretracheal fascia, and the cricothyroid membrane.

Copyrighl@ UWodd AI nghts , , _

~ Ill ® 0
Block Time Rematntng: 00:01:41
TUTOR Feedback Suspend End Block

••
•• _,
o• []
t

• ~
~--

·~ If A ~x
11:38 PM
!I;; ~·)) 2/12/2019 ~
~ OJ X
1
2
=
-
Item 18of24
Questoon ld: 11841 Ia \='Mark
<J
Previous
[>
Next
?
Tut o ria l
4l
Lab Values
~
Notes
~
Calculator
,
Reverse Color
aiiAJ 0
Text Zoom
3 •
4 A 58-year-old airline captain comes to the office after he was found to have abnormal vision testing during a pilot medical certification
5 examination. The patient feels well and has no ocular symptoms. Vision testing shows normal visual acuity but moderate loss of peripheral vision
6 in both eyes. Results of the funduscopic examination are shown in the image below.
7
8
9
10
11
12
13
14
15
16
• 17

19
20
21
22
23
24
~
1
2
3
4
5
6
7
8
9
10
11
12
13
14
Other neurologic and cardiopulmonary examination findings are unremarkable. He is started on latanoprost eyedrops. The medication is most
15
likely to improve this patient's condition through which of the following mechanisms?
16
17

0 A. Decreased bicarbonate formation in ciliary body

19 0 B. Decreased formation of abnormal blood vessels in retina

20
0 c. Decreased secretion of aqueous humor
21
22 0 D. Increased outflow of aqueous humor

23 0 E. Vasoconstriction in ciliary body


24

Submit
~
1
2
3
4
5
6
7
8
9
10
11
12
13
14
Other neurologic and cardiopulmonary examination findings are unremarkable. He is started on latanoprost eyedrops. The medication is most
15
likely to improve this patient's condition through which of the following mechanisms?
16
• 17 A. Decreased bicarbonate formation in ciliary body [2%)

B. Decreased formation of abnormal blood vessels in retina (10%)


19
20 C. Decreased secretion of aq ueous humor [21%)
21
D. Increased outflow of aqueous humor [61%)
22
23 E. Vasoconstriction in ciliary body (4%)
24

Omitted
10/07/2018
~ 01 X
1
2
= Item 18of24 <J [> ? &I ~ ~ , ail!) 0
- Questoon ld: 11841 Ia \='Mark Previous Next Tut o ria l Lab Values Notes Calculator Reverse Color Text zoom
3
4
5
Glaucoma medications
6
7
8 Trabecular
9
10
11
12
13
14
15 Trabecular outflow
16 • Muscarinic agonists
• 17

Schlemm
19
20
21
22 Aqueous humor inflow
23 • Beta blockers
24 Uveoscleral outflow • Alpha-2 AR agonists
• Prostaglandin agonists • CAis
~
1
2
3
Uveoscleral outflow • Alpha-2 AR agonists
4
• Prostaglandin agonists • CAis
5
6
7 Alpha-2 AR agonists =alpha-2 adrenergic receptor agonists; CAis =carbonic anhydrase inhibitors.
8 <l>UWOIId
9 Aqueous humor is secreted by epithelial cells of the ciliary body into the posterior eye chamber. The fluid then flows through the pupil into the
10 anterior chamber to the iridocomeal angle. Here, it diffuses through a traberular network into Schlemm's canal (scleral venous sinus) and
11 subsequently drains into episcleral and conjunctival veins. A small amount of aqueous can also diffuse through the ciliary musde and adjacent
12 sclera (uveosderal pathway).
13
Open-angle glaucoma is characterized by increased intraocular pressure due to increased secretion or decreased outflow of aqueous humor.
14
It is a form of optic neuropathy and causes progressive loss of ganglion cell axons, which may be visualized as a pale optic disc and enlarged
15
optic cup (compared to normal retina). Symptoms evolve over decades, with progressive loss of peripheral visual fields.
16
17
Glaucoma is managed with agents that either decrease production or increase outflow of aqueous humor. Latanoprost is a topical prostaglandin

used in the treatment of glaucoma. It is applied as a prod rug and converted to the active form by esterases in the cornea. Prostaglandins are the
preferred first-line agents for the treatment of glaucoma, and although the exact mechanism of action is uncertain, they have been found to
19
decrease the collagen content in the uveoscleral outflow pathway and increase outflow of aqueous hu mor. They are also known to cause
20
increased pigmentation in the iris and eyelashes .
21
22 (Choices A, C, and E) Carbonic anhydrase inh ibitors (eg, dorzolamide) decrease formation of bicarbonate in the ciliary body, lead ing to
23 decreased sodium and fluid transport and subsequently decreased prod uction of aqueous humor. Alpha-adrenergic agonists {eg, brimonidine)
24 inhibit production of aqueous humor due to vasoconstriction in the ciliary body. Beta blockers {eg, timolol) also decrease production of aqueous
humor. Prostaglandins do not affect production of aqueous humor but are often used in combination with these other agents.

(Choice B) Increased formation of new blood vessels at the optic disc is the characteristic feature of proliferative diabetic retinopathy. Strict
control of diabetes will reduce the risk of diabetic retinopathy, but it is not affected by topical prostaglandins.

~ Ill ® 0
Block Time Rematntng: 00:02:08
TUTOR Feedback Suspend End Block

••
•• o_,·
~ ·

[]
t

• ~
~--

·~ If A ~· ri?. ~·))
11:39 PM
2/12/Z019 ~
~ X
1
2
=
-
Item 18of24
Questoon ld: 11841 Ia \='Mark
<J
Previous
[>
Next
?
Tut o ria l
4l
Lab Values
~
Notes
~
Calculator
,
Reverse Color
aiiAJ
Text Zoom
0
3 Open-angle glaucoma is characterized by increased intraocular pressure due to increased secretion or decreased outflow of aqueous humor.
4 It is a form of optic neuropathy and causes progressive loss of ganglion cell axons, which may be visualized as a pale optic disc and enlarged
5 optic cup (compared to normal retina). Symptoms evolve over decades, w ith progressive loss of peripheral v isual fields.
6
Glaucoma is managed with agents that either decrease production or increase outflow of aqueous humor. Latanoprost is a topical prostaglandin
7
used in the treatment of glaucoma. It is applied as a prodrug and converted to the active form by esterases in the cornea. Prostaglandins are the
8
preferred first-line agents for the treatment of glaucoma, and although the exact mechanism of action is uncertain, they have been found to
9
decrease the collagen content in the uveosderal outflow pathway and increase outflow of aqueous humor. They are also known to cause
10
increased pigmentation in the iris and eyelashes.
11
12 (Choices A, C, and E) Carbonic anhydrase inhibitors (eg, dorzolamide) decrease formation of bicarbonate in the ciliary body, leading to

13
decreased sodium and fluid transport and subsequently decreased production of aqueous humor. Alpha-adrenergic agonists {eg, brimonidine)
inhibit production of aqueous humor due to vasoconstriction in the ciliary body. Beta blockers (eg, timolol) also decrease production of aqueous
14
humor. Prostaglandins do not affect production of aqueous humor but are often used in combination with these other agents.
15
16 (Choice B) Increased formation of new blood vessels at the optic disc is the characteristic feature of proliferative diabetic retinopathy. Strict
• 17 control of diabetes will reduce the risk of diabetic retinopathy, but it is not affected by topical prostaglandins.

Educational objective :
19
Glaucoma is a form of optic neuropathy characterized by increased intraocular pressure associated with increased production or decreased
20 outflow of aqueous humor. Topical prostaglandins (eg, latanoprost) increase outflow of aqueous via the uveoscleral pathway and are the preferred
21 treatment for open-angle glaucoma.
22
23
References
24
• Latanoprost ophthalmoc solution in the treatment of open angle glaucoma or raised intraocular pressure: a review.

Copyright® UWOI1d All ngllts reserved


19
20
21
22
23
24

Zoom In Zoom Out Reset


~
1
2
3
4
5
6
7
8
9
10
11
12
13
14
15
16
17

19
20
21
22
23
24

Zoom In Zoom Out Reset


~ OJ X
1
2
=
-
Item 19of 24
QuestiOn ld: 8557 a'? Mark
<J
Previous
[>
Next
?
Tutorial
4l
Lab Values
~
Notes
~
Calculator
,
Reverse Color
aiiAJ
Text Zoom
0
3
4 A 34-year-old woman comes to the physician complaining of double vision. She first experienced difficulty focusing her eyes while at w ork
5 yesterday. Upon waking up this morning, her symptoms had progressed to frank diplopia. Her past medical history is insignificant A complete
6 physical examination is performed, induding a full neurologic assessment. Shining light into her right eye causes constriction of her right pupil, but
7 not the left pupil. Shining light into her left eye causes constriction of only her right pupil. Which of the following additional physical examination
8 findings is likely to be found in this patient?
9
10
0 A. Absence of the left corneal reflex

11 0 B. Drooping of the left eyelid


12
13
0 C. Inability to close the left eye

14 0 D. Inward deviation of the left eye


15
0 E. Visual loss in the left eye
16
17
18
Submit

20
21
22
23
24

~ Ill ® 0
Block Time Rematntng: 00:02:09
TUTOR Feedback Suspend End Block

••
•• -·
~ ·
o· []
t

• ~
~·-
I~ if A

11:39 PM
t?i; ~·)) 2/12/2019 ~
~ OJ X
1
= Item 19of24 <J [> ? &I ~ ~ , ail!) 0
2 - QuestiOn ld: 8557 a'? Mark Previous Next Tutoria l Lab Values Notes Calculator Reverse Color Text Zoom
3 •
4 A 34-year-old woman comes to the physician complaining of double vision. She first experienced difficulty focusing her eyes while at work
5 yesterday. Upon waking up this morning, her symptoms had progressed to frank diplopia. Her past medical history is insignificant A complete
6 physical examination is performed, induding a full neurologic assessment. Shining light into her right eye causes constriction of her right pupil, but
7 not the left pupil. Shining light into her left eye causes constriction of only her right pupil. Which of the following additional physical examination
8 findings is likely to be found in this patient?
9
A. Absence of the left corneal reflex [ 17%1
10
11 B. Drooping of the left eyelid [60%1
12
C . Inability to close the left eye [3%1
13
14 D. Inward deviation of the left eye [10%1
15
E. Visual loss in the left eye [8%1
16
17
18
Omttted fi\ 3 Seconds ~ 12/ 19/2018
Correct answer "'' 60%
I.!!.!.
20 Answered correcUy \.::J Time Spent EE3 Last Updated
B
21
22
23 Explanation
24
Pupillary light reflex
~ 01 X
1
2
= Item 19of24 <J [> ? &I ~ ~ , ail!) 0
- Questoon ld: 8557 Ia \='Mark Previous Next Tut o ria l Lab Values Notes Calculator Reverse Color Text zoom
3
4 Pupillary light reflex
5
6
7
8
9
10
11
12
13
14
15
16
17
18

20
21
22
23 Oculomotor------,,£.-------/1
24 nerve
~
1
2
3
4
5
6
7
8
9
10
11 Oculomotor
12
nerve
13
14
15
16
17
18

Edinger-Westphal nucleus
20
21 Pretecta I
22
The pupillary light reflex is assessed by shini ng light in an eye and obseNing the response in that eye (direct) as well as the opposite eye
23
(consensual). The optic neNe (CN II) is responsible for the afferent limb of this reflex; the oculomotor neNe (CN Ill) is responsible for the efferent
24
limb. In this patient, the right CN II and right CN Ill are functional as there was a direct response seen when light was shi ned into the right eye. In
addition, the left CN II is also functional as a consensual response occurred when light was shined into the left eye. However, constriction of the
left pupil was not seen in either circumstance. Thus, this patient's lesion most likely involves her left CN Ill.

. . - - .. . -- ..

~ Ill ® 0
Block Time Remammg: 00:02:11
TUTOR Feedback Suspend End Block

••
•• _,
o• []
t

• ~
~--

·~ If A

11 :39 PM
!I;; ~·)) 2/12/2019 ~
~ X
1
2
=
-
Item 19of 24
QuestiOn ld: 8557 a'? Mark
<J
Previous
[>
Next
?
Tutorial
4l
Lab Values
~
Notes
~
Calculator
,
Reverse Color
aiiAJ
Text Zoom
0
3
The pupillary light reflex is assessed by shining light in an eye and observing the response in that eye (direct) as well as the opposite eye
4
(consensual). The optic nerve (CN II) is responsible for the afferent limb of this reflex; the oculomotor nerve (CN Ill) is responsible for the efferent
5
limb. In this patient, the right CN II and right CN Ill are functional as there was a direct response seen when light was shined into the right eye. In
6
addition, the left CN II is also functional as a consensual response occurred when light was shined into the left eye. However, constriction of the
7 left pupil was not seen in either circumstance. Thus, this patient's lesion most likely involves her left CN Ill.
8
The symptoms of CN Ill palsy relate to the function of CN Ill:
9
10 1. A somatic component innervates the inferior, superior, and medial rectus; inferior oblique; and levator palpebrae muscles. Ptosis occurs due
11 to paralysis of the levator palpebrae, and the unopposed action of the lateral rectus (CN Vl) and superior oblique (CN IV) musdes leads to a
12 "down-and-our gaze.
13 2. Parasympathetic fibers of CN Ill innervate the iris sphincter and ciliary musde. Paralysis of these fibers causes a fixed, dilated pupil and
14 loss of accommodation.

15 CN Ill palsy can result from lesions anywhere along the nerve's path from the oculomotor nucleus in the midbrain to the extraocular muscles within
16 the orbit. The most dreaded cause of isolated acute third nerve palsy is an actively enlarging intracranial aneurysm (at risk of imminent rupture).
17
18
(Choice A) Absence of the corneal reflex can result from lesions involving CN v, (afferent limb) or CN VII (efferent limb).
(Choice C) A lesion involving the left CN VII would resull in inability to close the left eye.
20
(Choice D) Inward deviation of the left eye would result from a CN VI lesion and the resultant unopposed action of CN Ill.
21
(Choice E) The left C N II is functional as it was able to produce a consensual response in the right eye.
22
23 Education01l objective :
24 The pupillary light reflex is assessed by shining light in an eye and observing the response in that eye (direct) and the opposite eye (consensual).
The optic nerve (CN II) is responsible for the afferent limb of the pupillary reflex, and the oculomotor nerve (CN Ill) is responsible for the efferent
limb.

Copyright ® UWor1d AI ngllls tese<Ved

~ Ill ® 0
Block Time Rematntng: 00:02:11
TUTOR Feedback Suspend End Block

••
•• -·
~ ·
o· []
t

• ~
~--
I~ if A

11 :40 PM
t?i; ~·)) 2/12/20 19 ~
~ OJ X
1
2
=
-
ltem20of 24
QuestiOn ld: 1356 a'? Mark
<J
Previous
[>
Next
?
Tutorial
4l
Lab Values
~
Notes
~
Calculator
,
Reverse Color
aiiAJ
Text Zoom
0
3
4
A 65-year-old male complains of bilateral visual difficulty that has progressed over the last year. Funduscopy reveals small yellow retinal lesions
5
clustered in the macula. Which of the following would you expect most on visual field examination?
6
7 0 A. Homonymous hemianopsia
8
9 0 B. Binasal hemianopsia

10 0 c. Bitemporal hemianopsia
11
12
0 D. Arcuate scotomas

13 0 E. Central scotomas

14
15
16 Submit
17
18
• 19

21
22
23
24

~ Ill ® 0
Block Time Rematntng: 00:02:12
TUTOR Feedback Suspend End Block

••
•• -·
~ ·
o· []
t

• ~
~·-
I~ if A

11:40 PM
t?i; ~·)) 2/12/2019 ~
~ OJ X
1
2
=
-
ltem20of24
QuestiOn ld: 1356 a'? Mark
<J
Previous
[>
Next
?
Tutoria l
4l
Lab Values
~
Notes
~
Calculator
,
Reverse Color
aiiAJ 0
Text Zoom
3 •
4
A 65-year-old male complains of bilateral visual difficulty that has progressed over the last year. Funduscopy reveals small yellow retinal lesions
5
clustered in the macula. Which of the following would you expect most on visual field examination?
6
7 A. Homonymous hemianopsia [2%)
8
B. Binasal hemianopsia (1 %)
9
10 C. Bitemporal hemianopsia (5%)
11
D. Arcuate scotomas (7%)
12
13 E. Central scotomas (81 %)

14
15
16 .......,tt1ed
17 Correct answer
111. 81% fl\ 3 Seconds ~ 02/06/2019
l!!!. Answered corredly '\.::.) Time Spent EH3 Last Updated
18 E

• 19

Explanation
21
22
The macula is a yellowish spot approximately 1.5 mm in diameter located near the center of the retina. It is characterized histologically by the
23
presence of densely packed cones, few overlying cells and no blood vessels. Each macular cone synapses to a single bipolar cell, which, in turn ,
24
synapses to a single ganglion cell. Due to this arrangement the visual acuity in the macula, and particularly the fovea, is greater than in any other
area of the retina. The neural fibers that serve the macula transmit to an area of the occipital visual cortex that is separate from the area of
representation of the peripheral fields (this area is also relatively large in size). Due to this peculiar cortical representation, macular sparing is
~ OJ X
1
2
= ltem20of24
a'?
<J [> ? 4l ~ ~ , aiiAJ 0
3
-
. ..
QuestiOn ld: 1356
.. ..
Mark Previous Next Tutorial Lab Values Notes Calculator Reverse Color Text Zoom

4
5 The macula is a yellowish spot approximately 1.5 mm in diameter located near the center of the retina. It is characterized histologically by the
6 presence of densely packed cones, few overlying cells and no blood vessels. Each macular cone synapses to a single bipolar cell, which, in turn,
7 synapses to a single ganglion cell. Due to this arrangement the visual acuity in the macula, and particularly the fovea, is greater than in any other
8 area of the retina. The neural fibers that serve the macula transmit to an area of the occipital visual cortex that is separate from the area of
9 representation of the peripheral fields (this area is also relatively large in size). Due to this peculiar cortical representation, macular sparing is
10 common in lesions of the occipital cortex.

11 Macular lesions impair central vision and result in central scotomata (Choice E). The term scotoma refers to any visual defect surrounded by a
12 relatively unimpaired field of vision. Scotomas occur due to pathologic processes that involve parts of the retina or optic nerve. Examples of such
13 processes include demyelinating diseases such as multiple sclerosis, diabetic retinopathy and retinitis pigmentosa. Pathologic processes that
14 involve the entire optic nerve lead to monocular blindness.
15
The patient described in this clinical vignette is likely to have macular degeneration (MD). This disorder is frequently age-related, and is the most
16
common cause of blindness in people over 50 years old in the US. Macular degeneration is characterized by progressive loss of central vision
17
due to deposition of fatty tissue (drusen) behind the retina (dry MD) and neovascularization of the retina (wet MD).
18
19
(Choice A) Homonymous hemianopsia is a loss of vision in the same side of the visual field in both eyes. It occurs due to transection of the

contralateral optic tract, the fibers of the visual pathway li nking the optic chiasm to the lateral geniculate body.

21 {Choice B) Binasal hemianopsia is a loss of vision in the nasal fields of both eyes. It is caused by pressure to the lateral areas of the optic
22 chiasm. This may result from calcified carotid arteries.
23
{Choice C) Bitemporal hemianopsia is a loss of vision in the temporal fields of both eyes. It is caused by compression of the medial part of the
24
optic chiasm, where the nasal fibers decussate, most commonly by a pituitary adenoma.

{Choice D) Arcuate scotomas occur due to damage to a particular region of the optic nerve head. The resulting visual field defect follovts the
arcuate shape of the nerve fiber pattern.

~ Ill ® 0
Block Time Rematntng: 00:02:14
TUTOR Feedback Suspend End Block

••
•• -·
~ ·
o· []
t

• ~
~--
I~ if A

11:40 PM
t?i; ~·)) 2/ 12/20 19 ~
~
1
2
3 an area
4 representation of the peripheral fields (this area is also relatively large in size). Due to this peculiar cortical representation, macular sparing is
5 common in lesions of the occipital cortex .
6
Macular lesions impair central vision and result in central scotomata (Choice E). The term scotoma refers to any visual defect surrounded by a
7 relatively unimpaired field of vision. Scotomas occur due to pathologic processes that involve parts of the retina or optic nerve. Examples of such
8 processes include demyelinating diseases such as multiple sderosis, diabetic retinopathy and retinitis pigmentosa . Pathologic processes that
9 involve the entire optic nerve lead to monocular blindness.
10
The patient described in this clinical vignette is likely to have macular degeneration (MD). This disorder is frequently age-related, and is the most
11
common cause of blindness in people over 50 years old in the US. Macular degeneration is characterized by progressive loss of central vision
12
due to deposition of fatty tissue (drusen) behind the retina (dry MD) and neovascularization of the retina (wet MD).
13
14 (Choice A} Homonymous hemianopsia is a loss of vision in the same side of the visual field in both eyes. It occurs due to transection of the
15 contralateral optic tract, the fibers of the visual pathway linking the optic chiasm to the lateral geniculate body.

16 (Choice B) Binasal hemianopsia is a loss of vision in the nasal fields of both eyes. It is caused by pressure to the lateral areas of the optic
17 chiasm. This may result from calcified carotid arteries.
18
(Choice C) Bitemporal hemianopsia is a loss of vision in the temporal fields of both eyes. It is caused by compression of the medial part of the
• 19
optic chiasm, where the nasal fibers decussate, most commonly by a pituitary adenoma.

21 (Choice D) Arcuate scotomas occur due to damage to a particular region of the optic nerve head. The resulting visual field defect follo\"15 the
22 arcuate shape of the nerve fiber pattem.

23 Educational Objective:
24 A scotoma is a visual field defect that occurs due to a pathologic process that involves parts of the retina or the optic nerve resulting in a discrete
area of altered vision surrounded by zones of normal vision. Lesions of the macula cause central scotomas.

Copyright® UWOI1d All ngllts reserved


~ OJ X
1
2
=
-
Item 21 of24
Questoon ld: 11183 Ia \='Mark
<J
Previous
[>
Next
?
Tutorial
4l
Lab Values
~
Notes
~
Calculator
,
Reverse Color
aiiAJ
Text Zoom
0
3
4 A 6-year-old boy is brought to the emergency department by his parents due to persistent nasal bleeding. The boy picks his nose frequently and
5 has had several nosebleeds in the past, all of which stopped spontaneously after pinching the nose. The parents say that they have been
6 pinching the nasal alae for over 30 minutes while the boy leans forward. Family history is negative for bleeding disorders. The patient takes no
7 medications and has no allergies. Examination shows continuous blood trickle from his right nostril. Silver nitrate cautery is performed and the
8 bleeding stops. Cautery was most likely applied to which of the following locations in this patient's nasal cavity?
9
10
O A. Inferior turbinate

11 0 B. Middle meatus
12
13
0 c. Nasal septum

14 0 D. Posterior choanae
15
16
0 E. Posterolateral wall

17
18
Submit
19
20

22
23
24

~ Ill ® 0
Block Time Rematntng: 00:02:15
TUTOR Feedback Suspend End Block

••
•• -·
~ ·
o· []
t

• ~
~·-
I~ if A

11 :40 PM
t?i; ~·)) 2/12/20 19 ~
~ OJ X
1
2
= Item 21 of24 <J [> ? &I ~ ~ , ail!) 0
- Questoon ld: 11183 Ia \='Mark Previous Next Tutorial Lab Values Notes Calculator Reverse Color Text Zoom
3 •
4 A 6-year-old boy is brought to the emergency department by his parents due to persistent nasal bleeding. The boy picks his nose frequently and
5 has had several nosebleeds in the past, all of which stopped spontaneously after pinching the nose. The parents say that they have been
6 pinching the nasal alae for over 30 minutes while the boy leans forward. Family history is negative for bleeding disorders. The patient takes no
7 medications and has no allergies. Examination shows continuous blood trickle from his right nostril. Silver nitrate cautery is performed and the
8 bleeding stops. Cautery was most likely applied to which of the following locations in this patient's nasal cavity?
9
A. Inferior turbinate [12%]
10
11 B. Middle meatus [10%]
12
c. Nasal septum [45%]
13
14 D. Posterior choanae [10%]
15
E. Posterolateral wall [21 %]
16
17
18
19 Om1tted llo. fi\ 3 Seconds
45% ~ 11 /20/2018
Correct answer l!!.!. Answered correcUy \.::J Time Spent EE3 Last Updated
20 c

22
23 Explanation
24
Kiesselbach's plexus

0 •

~ Ill ® 0
Block Time Rematntng: 00:02:17
TUTOR Feedback Suspend End Block

••
•• _,
o• []
t

• ~
~·-

·~ If A

1UOPM
!I;; ~·)) 2/12/2019 ~
~ 01 X
1
2
= Item 21 of24 <J [> ? &I ~ ~ , ail!) 0
- Questoon ld: 11183 Ia \='Mark Previous Next Tut o ria l Lab Values Notes Calculator Reverse Color Text zoom
3
4
Kiesselbach's plexus
5
6
7 Frontal sinus
8
9 Anterior ethmoid artery
10
Posterior ethmoid artery
11
12 Sphenopalatine artery
13
14 Sphenoid sinus
15
16
17
18 Kiesselbach's
19 plexus
20

22
23
24

Superior
labial artery
Sphenopalatine artery

Sphenoid sinus

22
USMLEWorld, LLC © 2011
Greater palatine artery
23
24
The nasal mucosa is highly vascular and easily irritated by trauma (eg, nose-picking), mucosal dryness, foreign body insertion, and rhinitis (eg ,
allergy, infection). Epistaxis is very common in children and may be classified as anterior or posterior, depending on the bleeding source.
Anterior nosebleeds are by far the most common, and the vast majority occur within the vascular watershed area of the nasal septum
(anteroinferior part of the nasal septal mucosa) known as Ki essel bach pl exus. Anastomosis of the following vessels occurs in this region:

~ Ill ® 0
Block Time Rematntng: 00:02:17
TUTOR Feedback Suspend End Block

••
•• _,
o• []
t

• ~
~--

·~ If A ~x
1UOPM
!I;; ~·)) 2/12/2019 ~
~ OJ X
1
2
= Item 21 of24 <J [> ? 4l ~ ~ , ail!) 0
- Questoon ld: 11183 Ia \='Mark Previous Next Tut o ria l Lab Values Notes Calculator Reverse Color Text Zoom
3 Greater palatine artery
USMLEWorld, LLC © 2011
4
5 The nasal mucosa is highly vascular and easily irritated by trauma (eg, nose-picking), mucosal dryness, foreign body insertion, and rhinitis (eg ,

6 allergy, infection). Epistaxis is very common in children and may be classified as anterior or posterior, depending on the bleeding source.

7 Anterior nosebleeds are by far the most common, and the vast majority occur within the vascular watershed area of the nasal septum
(anteroinferior part of the nasal septal mucosa) known as Ki esselbach pl exus. Anastomosis of the following vessels occurs in this region:
8
9 • Septal branch of the anterior ethmoidal artery
10 • Lateral nasal branch of the sphenopalatine artery
11 • Septal branch of the superior labial artery (branch of the facial artery)
12
Management is directed at stopping the bleeding from Kiesselbach plexus, preferably by direct compression of the nasal alae. Cautery (eg,
13 silver nitrate) of the area surrounding the bleeding site may be necessary for persistent bleeding.
14
(Choices A and B) The lateral nasal wall contains the superior, middle, and inferior turbinates (also known as conchae). These 3 bony
15
projections are covered with mucous membrane; they w arm, humidify, and filter inspired air and expand and contract in response to environmental
16
changes (eg, temperature, humidity, allergens). The turbinates form corresponding meatuses that serve as drainage pathways. The superior
17
meatus provides drainage for the sphenoidal and posterior ethmoidal sinuses. The middle meatus drains the frontal, maxillary, and anterior
18
ethmoidal sinuses and is the most common site of nasal polyps. The inferior meatus drains the nasolacrimal duct.
19
20 (Choices D and E) Branches of the sphenopalatine artery supply the posterolateral wall and posterior choanae. These posterolateral branches
are the most likely source of posterior nosebleeds. Posterior epistaxis is usually severe and cannot be treated with cautery.
22 Educational objective :
23 Epistaxis is commonly caused by irritation of the highly vascular mucosa at the anterior nasal septum. The anterior nasal septum contains the
24 Kiesselbach plexus. The anterior ethmoidal, sphenopalatine, and superior labial arteries anastomose in this region.

References
~ OJ X
1
= Item 21 of24 = <] C> ? 4f ~ ~ , ~ 0
2 - Question ld: 11783 ,. ? Mark Previous Next Tutorial Lab Values Notes Calculator Reverse Color Text zoom
3
4
5
How to stop epistaxis
6
7
8
9
10
11
Nasal bone
12
13
14
15
16
17
18
19
20

22
Kiesselbach
23 plexus
24

Zoom In Zoom Out Reset Add To Flash Card


~ OJ X
1
2
= ltem22of 24 <J [> ? &I ~ ~ , ail!) 0
- Questoon ld: 1633 Ia \='Mark Previous Next Tut o ria l Lab Values Notes Calculator Reverse Color Text Zoom
3 •
4 A 56-year-old previously healthy man comes to the office due to decreased hearing in both ears. He reports difficulty understanding conversations
5 in crowded rooms. His wife adds that they often argue about the volume of the television set. The patient cannot remember precisely when he
6 first noticed hearing loss but says it has been present at least 6 months and is getting worse. For the past 12 years, he has worked in a factory
7 where he has to shout to communicate with coworkers and has seldom worn hearing protection. An audiogram is obtained as shown in the image
8 below.
9
10 Frequency (Hz)
11
~ ~ ~ ~ ~ ~ ~
12
~1> <o\;:)\;:)'\<o\;:) ~\;:)(;:) ~<o(;S ~(;:) c;\;:)(;:) ~(;:) ro\;:)(;5 r:o\;:)(;5
13
-10
14
15
0
16
.....-..
17 0>
<.0 10°X 0 0
18 0> X
~
19
20 -Cf)- 20
21
z<(
••
30
23 Q)
24 0:::
...._... 40
co
'"0
c 50
~ 01 X
1
2
= ltem22of 24 <J [> ? &I ~ ~ , ail!) 0
- Questoon ld: 1633 Ia \='Mark Previous Next Tutorial Lab Values Notes Calculator Reverse Color Text zoom
3
0 •
4
5
6
-
0'>
<D
0'>
10°X 0
X
0
X
"(""""
7
8 -Cf)- 20 0
9
z
10 <{ 30
• •
11
<1> 0
12 0:: 0 X
13
'-"' 40
X
14
co
'0
15 c 50
16
17
-
·-
......1
I 60
X

18 '-"'
19 -<1>
20 > 70
<1>
21
0>
c 80
23 ·-' -
ro 90
24 <1>
I
100

~ Ill ® 0
Block Time Remaonong: 00:02:27
TUTOR Feedback Suspend End Block

••
•• _,
o• []
t

• ~
~--

·~ If A ~x
1U1PM
!I;; ~·)) 2/12/2019 ~
~
1
2
3
4 • •
Q)
5
6
7
-co
0::

"0
40

8 c 50
9
-
·-
_J
60
X

--
10
I
11
12 Q)
13 > 70
Q)
14
15
0>
c 80
16
·-' -
ro 90
17 Q)
18 I
19 100
20
21
110
23 © UWorld 0 Right ear air conduction x Right ear bone conduction
24
Which of the following is most likely abnormal in this patient?

Q A. Auditory nerve
-· B. Cochlear cu ula
Block Time Remaining : 00:02:32
TUTOR ~ IJI
Feedback
®
Suspend
0
End Block

•• -o·
_,· lfi w -• rGJ rc:"~
11 :41 PM
•• IJ9 •• >f A ~? x ~
ft?. <l>l) 2/12/2019 ~
~ 01 X
1
2
= ltem22of 24 <J [> ? &I ~ ~ , ail!) 0
- Questoon ld: 1633 Ia \='Mark Previous Next Tutorial Lab Values Notes Calculator Reverse Color Text zoom
3
4
c0> 80
5
6
·-" -
7
~ 90
8
I
9 100
10
11 110
12
13 ©UWorfd 0 Right ear air conduction x Right ear bone conduction
14 Which of the following is most likely abnormal in this patient?
15
16 O A. Auditory nerve
17
0 B. Cochlear cupula
18
19 0 C. Middle ear ossicles

20 0 D. Organ of Corti
21
0 E. Round window

23 O F. Tympanic membrane
24

Submit
~ OJ X
1
2
= ltem22of 24 <J [> ? &I ~ ~ , ail!) 0
- Questoon ld: 1633 Ia \='Mark Previous Next Tutorial Lab Values Notes Calculator Reverse Color Text zoom
3
4
c0> 80
5
6
·-" -
7
~ 90
8
I
9 100
10
11 110
12
13 ©UWorfd 0 Right ear air conduction x Right ear bone conduction
14 Which of the following is most likely abnormal in this patient?
15
16 A. Auditory nerve [10%]
17 B. Cochlear cupula [13%]
18
C. Middle ear ossicles [13%]
19
20 D. Organ of Corti (51%]
21
E. Round window (2%]

23 F. Tympanic membrane [8%]


24

Omitted
~ 01 X
1
2
= ltem22of 24 <J [> ? &I ~ ~ , ail!) 0
- Questoon ld: 1633 Ia \='Mark Previous Next Tut o ria l Lab Values Notes Calculator Reverse Color Text zoom
3
4 Organ of corti
5
6
7
8
9
10
11
Oval window
12
13
14
15
16
17 Scala tympani
18
Cochlear duct
19
20 Tectorial membrane
21

23
24
~
1
2
3
4
Cochlear duct

5
6
7
8
9
10
11
12
13
14 Cochlear nerve fibers
Cl UWo~d
15
16
This patient has high-frequency hearing loss due to chronic noise exposure. Transduction of mechanical sound waves into nerve impulses occurs
in the organ of Corti through the following steps:
17
18 1. Sound reaches the middle ear by vibrating the tympanic membrane.
19 2. The vibration is transferred to the oval window by the ossicles.
20 3. Vibration of the oval window causes vibration of the basilar membrane, which in turn causes bending of the hair cell cilia against the tectorial
21 membrane.
4. Hair cell bending causes oscillating hyperpolarization and depolarization of the auditory nerve, thereby creating nerve impulses from sound .
23 Noise-induced hearing loss results from trauma to the stereociliated hair cells of the organ of Corti. The acoustic reflex normally dampens the
24 effects of loud noise by causing the stapedius and tensor tympani muscles to contract, which lessens the responsiveness of the ossicles to sound.
However, prolonged noise exposure can cause distortion or fracture of the stereocilia due to shearing forces against the tectorial membrane.
High-frequency hearing is lost first, regardless of the frequency of the sound causing the damage.
~ OJ X
1
2
=
-
ltem22of 24
Questoon ld: 1633 Ia \='Mark
<J
Previous
[>
Next
?
Tutorial
4l
Lab Values
~
Notes
~
Calculator
,
Reverse Color
aiiAJ
Text Zoom
0
3
1. Sound reaches the middle ear by vibrating the tympanic membrane.
4
2. The vibration is transferred to the oval window by the ossid es.
5
3. Vibration of the oval window causes vibration of the basilar membrane, which in turn causes bending of the hair cell cilia against the tectorial
6
membrane.
7
4. Hair cell bending causes oscillating hyperpolarization and depolarization of the auditory nerve, thereby creating nerve impulses from sound.
8
9 Noise-induced hearing loss results from trauma to the stereociliated hair cells of the organ of Corti. The acoustic reflex normally dampens the
10 effects of loud noise by causing the stapedius and tensor tympani muscles to contract, which lessens the responsiveness of the ossicles to sound.
11 However, prolonged noise exposure can cause distortion or fracture of the stereocilia due to shearing forces against the tectorial membrane.

12
High-frequency hearing is lost first, regardless of the frequency of the sound causing the damage.

13 (Choice A) Hearing loss originating in the auditory nerve is most commonly due to a vestibular schwannoma (acoustic neuroma). Patients
14 develop high-frequency sensorineural hearing loss, but symptoms are usually unilateral.
15
(Choice B) The cochlear cupula is the apex of the cochlea and is distant from the oval window. The distal cochlea primarily registers low-
16
frequency sound.
17
18 (Choice C) Defects of the middle ear ossicles (eg, otosclerosis) can cause hearing loss that generally affects air conduction across all

19
frequencies. This patient's hearing loss affects both air and bone conduction.

20 (Choice E) The round window lies between the middle and inner ear and moves outward when the stapes causes the oval window to move
21 inward. Malformation of the round window can cause congenital hearing impai rment, but acquired disorders are rare.

(Choice F) Perforation or rupture of the tympanic membrane can occur with infection, trauma , pressure changes, or sudden and very loud
23
noises. Rupture of the tympanic membrane causes unilateral conductive hearing loss.
24
Educational objective:
Prolonged exposure to loud noises causes hearing loss due to damage to the stereociliated hair cells of the organ of Corti.

~ Ill ® 0
Block Time Rematntng: 00:02:39
TUTOR Feedback Suspend End Block

••
•• -·
~ ·
o· []
t

• ~
~--
I~ if A

11 :4 1 PM
t?i; ~·)) 2/12/20 19 ~
~ OJ X
1
= ltem23of24 <J [> ? 4l ~ ~ , ail!) 0
2 - QuestiOn ld: 1698 a'? Mark Previous Next Tutoria l Lab Values Notes Calculator Reverse Color Text Zoom
3 •
4 A 30-year-old man comes to the emergency department due to left eye pain after being involved in a street fight. He received a fist blow to the left
5 eye and has since had pain and diplopia. Past medical history is notable for 2 previous gunshot wounds. The patient does not smoke cigarettes
6 but has a history of heavy alcohol intake and frequent marijuana use. On examination , there is significant soft tissue swelling around the left eye.
7 Visual acuity is normal, but extraocular motility of the left eye is limited. Pupillary reflexes are normal. Coronal CT scan of the orbits is shown in
8 the image below.
9
10
11
12
13
14
15
16
17
18
19
20
21
22
24
~
1
2
3
4
5
6
7
8
9
10 '
11
12
13
14
15
16


17
18
19
' '.
20
21
22 The orbital contents of this patient are most likely to herniate into which of the following sites?

24 0 A. Ethmoid cells

0 B. Frontal sinus

0 c. Inferior conchae

~ Ill ® 0
Block Time Remammg: 00:02:48
TUTOR Feedback Suspend End Block

••
•• _,
o• e mw []
t

• ~
~--

·~ If A ~x
1U1PM
!I;; ~·)) 2/12/2019 ~
~
1
2
3
4
5
6
7
8
9
10
11
12
13
14
15
16
17
18
19
20
21
22

24

Zoom In Zoom Out Reset Add To Flash Card

~ IJI ® 0
Block Time Remaining : 00:02:52
TUTOR Feedback Suspend End Block

•• -o·
_,· lfi w -• rGJ rc:"~
11 :41 PM
•• IJ9 •• >f A ~? x ~
ft?. <l>l) 2/12/2019 ~
~
1
2
3
4
5
6
7
8
9
10
11
12
13
14
15
The orbital contents of this patient are most likely to herniate into which of the following sites?
16
17
18
O A. Ethmoid cells

19 0 B. Frontal sinus

20
0 c. Inferior conchae
21
22 0 D. Maxillary sinus

0 E. Sphenoid sinus
24

Submit
~
1
2
3
4
5
6
7
8
9
10
11
12
13
14
15
The orbital contents of this patient are most likely to herniate into which of the following sites?
16
17 A. Ethmoid cells (4%]
18
19
B. Frontal sinus [2%]

20 c. Inferior conchae (0%]


21
D. Maxillary sinus [85%]
22
E. Sphenoid sinus (6%]
24

Omitted
11 /09/2018
~ OJ X
1
2
=
-
ltem23of24
QuestiOn ld: 1698 a'? Mark
<J
Previous
[>
Next
?
Tutoria l
4l
Lab Values
~
Notes
~
Calculator
,
Reverse Color
aiiAJ
Text Zoom
0
3
4 Omttted II\ 18 Seconds ~ 11/09/2018
Correct answer L!!!. 85%
'"·
5 Answered correcHy \.::) Ttme Spent BE Last Updated
D
6
7
8 Explanation
9
10 The orbit is bound superiorly by the thick orbital plate of the frontal bone and laterally by the thick bone of the zygoma and greater and Jesser
11 sphenoid wings. In contrast, the orbital floor is composed of a very thin layer of bone that separates the orbit from the air-filled maxillary sinus.
12 Similarly, the orbirs medial wall is composed of the thin ethmoid and lacrimal bones, which separate it from the ethmoid air cells.
13
Blunt trauma to the eye causes a rapid increase in pressure that typically does not rupture the globe but is transmitted posteriorly into the orbit.
14
The weakest plates of bone in the orbit, the medial and inferior walls, are common sites of fracture. Fracture is typically evident on radiographic
15
imaging, and fluid (eg, blood) or herniation of the orbital contents can often be visualized in the adjacent normally air-filled sinuses.
16
17 (Choice A) The ethmoid air cells are medial to the orbit. They are unaffected in this patient, but they can be affected in blowout fractures of the
orbit's medial wall.
18
19 (Choice B) The frontal sinus is not pictured above. It lies in the medial part of the supraorbital frontal bone.
20
(Choi ce C) The inferior nasal conchae form part of the nasal turbinate system. They are not air-filled sinuses.
21
22 (Choice E) The sphenoid sinuses (not pictured above) lie anterior to the optic chiasm and are not in close relation to the orbit.

Educational objective :
24 Blunt trauma to the globe can cause orbital blowout fractures. These fractures most commonly involve the medial or inferior orbital walls due to
the thin bone bordering the ethmoid and maxillary sinuses.

Copyright® UWOI1d All ngllts reserved


~ OJ X
1
= Item 23 of24 = <] C> ? 4f ~ ~ , ~ 0
2 - Question ld: 1698 ,. ? Mark Previous Next Tutorial Lab Values Notes Calculator Reverse Color Text zoom
3
4
5
6
7
8
9
10
11
12
13
14
15
16
17
18
19
20
21
22

24

Zoom In Zoom Out Reset Add To Flash Card

~ IJI ® 0
Block Time Remaining : 00:02:57
TUTOR Feedback Suspend End Block

•• -o·
_,· lfi w -• rGJ rc:"~
11 :41 PM
•• IJ9 •• >f A ~? x ~
ft?. <l>l) 2/12/2019 ~
~ OJ X
1
2
=
-
ltem24of 24
QuestiOn ld: 1686 a'? Mark
<J
Previous
[>
Next
?
Tutorial
4l
Lab Values
~
Notes
~
Calculator
,
Reverse Color
aiiAJ
Text Zoom
0
3
4 A 44-year-old man comes to the office for evaluation of a neck lump. The patient first noticed the lump 6 months ago while he was shaving and
5 says that it seems to be growing in size. He has no pain, shortness of breath, cough, hoarseness, or difficulty swallowing. The patient had
6 Hodgkin lymphoma 20 years ago, which was successfully treated with radiation therapy to his chest and neck. Physical examination shows a firm
7 mass in the left thyroid lobe. There is no lymphadenopathy. Serum TSH level is normal. Fine-needle aspiration biopsy reveals papillary thyroid
8 cancer. A total thyroidectomy is performed. During an attempt to ligate the inferior thyroid artery, a nervous structure in close proximity is
9 inadvertently damaged. Which of the following was most likely injured during the surgery?

10
11
O A. Accessory nerve

12 0 B. Ansa cervicalis
13
14
0 C. Hypoglossal nerve

15 0 D. Phrenic nerve

16
17
0 E. Recurrent laryngeal nerve

18 0 F. Superior cervical ganglion

19
0 G. Superior laryngeal nerve
20
21
22 Submit
23

~ Ill ® 0
Block Time Rematntng: 00:02:58
TUTOR Feedback Suspend End Block

••
•• -·
~ ·
o· e mw []
t

• ~
~--
I~ if A ~·
11:42 PM
t?i; ~·)) 2/12/2019 ~
~ OJ X
1
2
=
-
ltem24of 24
QuestiOn ld: 1686 a'? Mark
<J
Previous
[>
Next
?
Tutorial
4l
Lab Values
~
Notes
~
Calculator
,
Reverse Color
aiiAJ
Text Zoom
0
3 •
4 A 44-year-old man comes to the office for evaluation of a neck lump. The patient first noticed the lump 6 months ago while he was shaving and
5 says that it seems to be growing in size. He has no pain, shortness of breath, cough, hoarseness, or difficulty swallowing. The patient had
6 Hodgkin lymphoma 20 years ago, which was successfully treated with radiation therapy to his chest and neck. Physical examination shows a firm
7 mass in the left thyroid lobe. There is no lymphadenopathy. Serum TSH level is normal. Fine-needle aspiration biopsy reveals papillary thyroid
8 cancer. A total thyroidectomy is performed. During an attempt to ligate the inferior thyroid artery, a nervous structure in close proximity is
9 inadvertently damaged. Which of the following was most likely injured during the surgery?

10
A. Accessory nerve [0%)
11
12 B. Ansa cervicalis [2%)
13
C. Hypoglossal nerve [0%)
14
15 D. Phrenic nerve [1 %]

16 E. Recurrent laryngeal nerve [81%)


17
F. Superior cervical ganglion [0%]
18
19 G. Superior laryngeal nerve [13%)
20
21
22
Ormlted
23 Correct answer '"· 81%
IT'\ 3 Seconds ~ 10/11/2018
1.!.!!. Answered correcUy \.:::) Time Spent EE3 Last Updated
E

Explanation

~ Ill ® 0
Block Time Rematntng: 00:03:00
TUTOR Feedback Suspend End Block

••
•• -·
~ ·
o· []
t

• ~
~--
I~ if A

11:42 PM
t?i; ~·)) 2/12/2019 ~
~
1
2
3
4
5
6
7
8
9
10
11
12
13
14
15
16
17
18
19
©UWorld
20
21 The inferior thyroid artery arises from the thyrocervical trunk, a branch of the subclavian artery. Along w ith the superior thyroid arteries

22 (branches of the extemal carotid arteries), the inferior thyroid arteries provide blood to the thyroid gland and other structures of the anterior neck.
From the thyrocervical trunk, the inferior thyroid artery courses deep to the internal jugular vein, vagus nerve, and common carotid artery before
23
turning toward the thyroid. As it approaches the thyroid, the artery comes in close proximity to the recurrent laryngeal nerve.

The recurrent laryngeal nerve is a branch of the vagus nerve that loops below the aortic arch on the left and below the subclavian artery on the
right to provide motor innervation to the intrinsic muscles of the larynx (except the cricothyroid}. During thyroid surgery, this nerve may be
damaged due to its proximity to the inferior thyroid artery. Unilateral nerve injury often causes hoarseness, whereas bilateral injury may cause

~ Ill ® 0
Block Time Rematntng: 00:03:00
TUTOR Feedback Suspend End Block

••
•• _,
o• []
t

• ~
~·-

·~ If A ~x
11o42 PM
!I;; ~·)) 2/12/2019 ~
~
1
2
3
4 The inferior t hyroid artery arises from the thyrocervical trunk, a branch of the subclavian artery. Along with the superior thyroid arteries
5 (branches of the external carotid arteries}, the inferior thyroid arteries provide blood to the thyroid gland and other structures of the anterior neck.
6 From the thyrocervical trunk, the inferior thyroid artery courses deep to the internal jugular vein, vagus nerve, and common carotid artery before
7 turning toward the thyroid. As it approaches the thyroid, the artery comes in clo se proximity to the recu rrent laryngeal nerve.
8
The recurrent laryngeal nerve is a branch of the vagus nerve that loops below the aortic arch on the left and below the subclavian artery on the
9
right to provide motor innervation to the intrinsic muscles of the larynx (except the cricothyroid). During thyroid surgery, this nerve may be
10
damaged due to its proximity to the inferior thyroid artery. Unilateral nerve injury often causes hoarseness, whereas bilateral injury may cause
11
inspiratory stridor and respiratory distress due to complete v ocal cord paralysis.
12
13 (Choice A) The spinal accessory nerve (CN XI) exits the jugular foramen and courses on top of the levator scapulae muscle to innervate the

14 sternocleidomastoid and trapezius muscles on either side.

15 (Choice B ) The ansa cervicalis is a loop of the cervical plexus that receives contributions from C1-C3. It courses deep to the sternocleidomastoid
16 muscle and loops around the internal jugular vein. Branches from the ansa cervicalis innervate the sternohyoid, sternothyroid, and omohyoid
17 muscles.
18
(Choice C) The hypoglossal nerve (CN XII) exits the cranium through the hypoglossal canal and innervates all the muscles of the tongue except
19
the palatoglossus.
20
21 (Choi ce D) The phren ic nerve arises from the C3-C5 cervical nerves and innervates the diaphragm. The most common cause of iatrogenic

22 phrenic nerve injury is cardiothoracic surgery, in which the nerve can be damaged where it passes between the pericardium and mediastinal
pleura. Manifestations include diaphragmatic paralysis (inspiratory weakness).
23
(Choice F) The superior cervical ganglion is part of the sympathetic nervous system. It can be injured by an expanding lung cancer at the
superior pulmonary sulcus (Pancoast tumor), leading to Horner syndrome (ie, ipsilateral ptosis, miosis, and anhydrosis).

(Choice G) The superior laryngeal nerve branches off the vagus nerve high in the neck and divides into an external branch, which supplies the
cricothyroid muscle, and an internal branch, which provides sensation over the area. The external branch of this nerve be

~ Ill ® 0
Block Time Remammg: 00:03:00
TUTOR Feedback Suspend End Block

••
•• o_,·
~ ·

[]
t

• ~
~--

·~ If A ~· ri?. ~·))
11 :42 PM
2/12/20 19 ~
~
1
2
3
4
sternocleidomastoid and trapezius muscles on either side.

5 (Choice B) The ansa cervicalis is a loop ofthe cervical plexus that receives contributions from C1-C3. It courses deep to the sternocleidomastoid
6 muscle and loops around the internal jugular vein. Branches from the ansa cervicalis innervate the sternohyoid, sternothyroid , and omohyoid
7 muscles.
8
(Choice C) The hypoglossal nerve (CN XII) exits the cranium through the hypoglossal canal and innervates all the muscles of the tongue except
9
the palatoglossus.
10
11 (Choice D) The phrenic nerve arises from the C3-C5 cervical nerves and innervates the diaphragm. The most common cause of iatrogenic

12
phrenic nerve injury is cardiothoracic surgery, in which the nerve can be damaged where it passes between the pericardium and mediastinal
pleura. Manifestations include diaphragmatic paralysis (inspiratory weakness).
13
14 (Choice F) The superior cervical ganglion is part of the sympathetic nervous system. It can be injured by an expanding lung cancer at the
15 superior pulmonary sulcus (Pancoast tumor), leading to Horner syndrome 0e, ipsilateral ptosis, miosis, and anhydrosis).
16
(Choice G) The superior laryngeal nerve branches off the vagus nerve high in the neck and divides into an external branch, which supplies the
17
cricothyroid muscle, and an internal branch, which provides sensation over the supraglottic area. The external branch of this nerve may be
18 misligated during thyroid surgery due to its proximity to the superior thyroid artery.
19
Educational objective :
20
The recurrent laryngeal nerve travels in close proximity to the inferior thyroid artery and can be injured during thyroid surgery, resulting in vocal
21
cord paralysis (eg, hoarseness).
22
23
References

• Anatom1cal relationship between recurrent laryngeal nerve and inferior thyroid artery m thyroidectomy patients.

• An applied anatom1cal study on the recurrent laryngeal nerve and inferior thyroid artery

~ Ill ® 0
Block Time Remammg: 00:03:00
TUTOR Feedback Suspend End Block

••
•• -·
~ ·
o· []
t

• ~
~·-
I~ if A

11:42 PM
t?i; ~·)) 2/12/2019 ~

Вам также может понравиться